(17 Nov) - C4 Seniors, November 22-2-2
(17 Nov) - C4 Seniors, November 22-2-2
(17 Nov) - C4 Seniors, November 22-2-2
November - 2022
Sources: Hunter + SMLE-B + Silent
Group link
https://t.me/C4Seniors
Highlights
Green: Corrected question (Search the word “corrected”)
Yellow: Unsolved question
Purple: Another recall
Black: Repeated Question, will be deleted soon (Search the word “repeated”)
1
1. Child examination: look to the wall , bend forward this examination of waht?
A. Scoliosis
B. Sexual abuse
C. rectum prolapse
Better recall: Doctor ask the child to face the wall and bend forward and hanging his
arms unsupported, this test for :
A. scoliosis
B. leg length symmetry
C. rectal prolapse
D. sexual abuse
Answer: A
Speciality: Surgery
Explanation: Amboss
2
2. pediatric pt come to ER with hx of tonic clonic seizure duration 3 min 1 hour ago
, hx of fever and runny nose 3 days ago. Every thing is normal even temp. Appropriate
Tx?
A. Paracetamol ,
B. diazepam,
C. phenytoin
Answer: A
Speciality: Pedia
Explanation: Dr. Safder
3
3. hematemesis with abdominal pain first time no previous symptoms no medical
hx no medication , all lab normal, dx?
A. Gastritis ,
B. peptic ulcer,
C. immune hepatitis,
D. mallory weiss
Answer: B
Speciality: Medicine
Explanation:
Not A: A presents with Dyspepsia, Postprandial fullness, early satiety, Bloating
Not C: AIH has an insidious onset in most patients and its presentation varies widely,
ranging from asymptomatic disease to severe symptoms or even acute liver failure.
Not D: No history of forceful or recurrent vomiting .. not alcoholic
4
5
4. Asking pregnant women with no remarkable history on pregnancy came on 34 -
37 not sure
The test results was ketone bodies positive and protein I think
BP and everything was normal
The option was
A. CS
B. observation
C. induction of L
Answer: C
Speciality: OB/GYN
Explanation: DR. WAFA
6
7
5. Child presented after 1 week of bloody diarrhoea with edema and renal
inpairment( labs given there is hypertension and impaired renal analysis)
Asking what culture should help in diagnosis
A. urin
B. blood
C. stool
D. LP
Answer: C
Speciality: Pedia
Explanation: Dr. Safder
8
6. 36 male presented with hypertension (high blood pressure reading given) he
have a family history of hypertension and both his sisters have hypertension
He have high ceriatinin and protein in urin
How will you manage?
A. ACEI
B. alpha blocker
C. beta blocker
D. ?
Answer: A
Speciality: Medicine
Explanation:
If they have sweating and headache, it'd be suggestive of Pheochromocytoma. If so,
give Alpha blocker
If only hypertension and nothing else, ACEi seems reasonable.
9
7. chile i don’t remember the age presented with diarrhoea and vomiting for 1 day
he looks lethargic and ill cry with tears capillary refill is 3 . Management? Complete q
in safdar
1-10% dextrose
2-ORS na:glucose 1:1
Answer: B
Speciality: Pedia
Explanation: 2 questions mixed together 😂
10
8. child came in 6 month vaccination mother said he have anaphylactic reaction
required hospital admission after 4 month vaccination but she don’t know what
vaccine
What should be done now?
A. do allergic test first
B. vaccination except for dtap
C. reassures and vaccinate
D. ?
Answer: A
Speciality: PEDIA
Explanation:
11
9. pf after PE stared on heparin ..labs show prolong PT and PTTT
A. stop and give …?!(i forget the drug name but start with L!)
B. continue
C. switch to theraputic dose of enoxaprin
D. stop and !!!
Better recall: Case of P.E and recieve heparin after days give labs All normal except
platelet 80 PTT elevate Wts next?
A. Stop heparin and give Lepirudin
B. Give LMW
C. Increse heparin
Answer: A
Speciality: Medicine
Explanation:
12
13
10. female after MVA admitted for observation was vitaly stable x ray showed
fracture in 3and7 rib …then she devolution shortness of breath (only) ..what is the
cause ?
A. flial chest
B. PE
C. tension pneumothorax
D. ?
Answer: C
Speciality: Surgery
Explanation: Amboss
Flail chest is defined as 3 or more adjacent ribs fractured in 2 or more places.
Here is only 2 ribs (3rd and 7th)
14
15
11. pregnant diagnosed placenta previa what is the most associated with it
A. post coital bleeding spotting
B. profuse bleeding at the first episode
C. -?
Answer: A
Speciality: OB/GYN
Explanation: Dr. Wafa
16
12. a case of child after brain surgery
Na177 i forget the exact lab for urin
A. central DI
B. SIADH
C. ?
Answer: A
Speciality: Medicine
Explanation: Amboss
17
13. diabatic pt and obese (i think bmi was 41 )failed to reduse weight
A. diabetic drug for reducing weight
B. bariatric surgery
Answer: B
Speciality: Medicine
Explanation: UpToDate
18
14. 6 years old child with long history of multiple lower respiratory infection and
rhinitis admitted with respiratory symptoms bronchoscope done, sputum collection
done and culture was positive for pneudomonus aergunosa and ? What is the cause
A. primary ciliary dyskinesia
B. cystic fibrosis
C. bronchial asthma
D. !
Answer: B
Speciality: Pedia
Explanation: Dr. Safder
19
15. female presented with left knee pain for 3 days there is swelling and fever
Labs showed slighty high wbc and ESR high
Joint aspiration showed leukocytosis i think 200
X ray left knee showed decreased joint space and osteophyte
Diagnosis
A. septic arthritis
B. osteoarthritis
C. gout
D. rheumatoid arthritis
Answer: A
Speciality: Medicine
Explanation: We asked “review team” they answered the question A
20
16. pregnant 12week have cervical lesion what to do
A. pap smear
B. cone biopsy
C. Colposcopy
D. ?
Answer: C
Speciality: Ob/Gyn
Explanation: Dr. Wafa
21
17. 43male have family history of colon cancer underwent sigmoidoscopy for polyp
removal Histopathology showed tubular adenoma completely removed ..how to
follow?
A. 3-6 month
B. 3years
C. 5 years
D. no need
Another recall: Patient did a colonoscopy 1 polyp size 1.5cm and a biopsy showed
tubular adenoma. Interval of screening?
A. 6 month
B. 3years
C. 5-10 years
D. no need
Answer: We have to know the size to decide. If it is 1.5 cm as in the previous recall
the answer is B (3 years)
Speciality: Surgery
Explanation: UpToDate
1.5 cm = 15 mm
22
23
18. Pregnant woman at known to have seizure disorder on phenytoin ﻣﺎ ﻛﺘﺒﻮ الGA
what to do about her medication?
A. switch the medication
B. continue the medication
C. decrease the dose
D. increase the dose
Another recall: Pregnant woman at 12 week gestation known to have seizure disorder
on phenytoin (last seizure attack 6 years ago), what to do about her medication?
A. switch the medication
B. continue the medication
C. decrease the dose
D. increase the dose
Answer: B
Speciality: OBGYNE
Explanation: Management of epilepsy during preconception, pregnancy, and the
postpartum period - UpToDate
24
19. 60somthing female presentes after long travel with car for 5 hours with SOB
A. pericarditis
B. PE
C. -?
Answer: B
Speciality: Medicine
Explanation: Amboss
25
20. Female pt with vaginal candida, which following disease she has?
A-DM
B- HTN
C- Epilepsy
Answer: A
Speciality: Ob/Gyn
Explanation: Amboss
26
21. 37 years old female her father has colon cancer when he was 55 and her mom
had breast cancer when she was 43, asking about screening?
A.She should do mammogram annually
B.Start mammogram at 40
C.Start mammogram at 40 and colonoscopy at 55
D.Colonoscopy at 50
Answer: A (Corrected)
Speciality: Surgery
Explanation: (MOH)
Colon cancer screening in her condition must be started at 40.
Since there is positive family hx, screening for breast cancer should be started
immediately
Also, SMLE surgery channel, Hamza, and Dr. Abeidi answered this question (A)
27
22. X ray Picture: Elderly with abdominal pain, and vomiting. On abdominal x ray
you see a cofffee been shape in the intestines with part of the large colon twisting to
the upper right hypochondirum what’s the diagnosis?
A. Sigmoid volvulus
Answer: A
Speciality: Surgery
Explanation:
28
23. Female patient came to ER complaining of RUQ pain from 8h, the patient has
jaundice with fever of 37.9
Elevated WBC.
A. pancreatitis
B. cholangitis
C. Cholecystitis
Answer: B
Speciality: Surgery
Explanation: Acute cholangitis - AMBOSS
29
24. Standard therapy for PPH due to uterine atony?
A-oxytocin
B-ergot
Answer: A
Speciality: Ob/Gyn
Explanation: Dr. Wafa
30
25. 37 year old female pt g8 p7 at 36 weeks of gestation had previous recurrent
postpartum that required blood transfusion.
She is medically free
What is the most appropriate preventable measure in delivery?
A- deliver the patient in Operation Room
B- elective cs in 38 week of gestation
C- start active management in 3rd stage of labor
D- give preload crystalloid fluid in labor to over come postpartum hypovolemia
Answer: C
Speciality: Ob/Gyn
Explanation:
31
26. Management of PROM in active herpes simplex virus
A CS
B IV acyclovir
Answer: A
Speciality: OBGYNE
Explanation: Genital herpes simplex virus infection and pregnancy - UpToDate
32
27. Case of PROM at 37 weeks when to give antibiotics?
A- prior to delivery
B- Postpartum
Answer: A
Speciality: OBGYNE
Explanation: Dr Wafa
33
28. Pregnant in 34 week of GA came with labor pain admitted, she had PROM 4
weeks ago , O/E cervix is 3 cm, there is clear liquor, Ask about next step?
A- Emergency CS
B- call Anastasia and deliver pt
C- Wait until 37 weeks
D- give her corticosteroids
Answer: B
Speciality: OBGYNE
Explanation: Dr Wafa
34
29. RTA one lung dull to percussion
A- tension pneumothorax
B- massive hemothorax
C- pulmonary contusion
Answer: B
Speciality: Surgery
Explanation: Pleural effusion - AMBOSS
35
30. Female had infertility for 6 years, complains of severe dysmenorrhea
A-Leiomyoma
B-Endometriosis
C-Adenomyosis
Answer: B
Speciality: Ob/Gyn
Explanation: UTD
36
31. Nullipaurs with menorrhagra and occasional dysmenorrhea want contraception
what to give
A- ocp
B-Depo injection
C- vaginal ring
D- progesterone only pills
Answer: A
Speciality: ObGyn
Explanation: (Dr. Wafa’a)
📝📌Note to remember
Estrogen-progestin oral contraceptives (OCs) are first-line management for AUB. Also, it's used for
dysmenorrhea.
37
32. 44 female c/o dysmenorrhea + menorrhage Pelvic exam revealed uterus
symmetric enlarge and tender endometrial biopsy normal
A- adenomosis
B- endometriosis
C- sarcoma
D- liomyoma
Answer: A
Speciality: ObGyn
Explanation: (Dr. Wafa’a)
📝📌Note to remember:
Adenomyosis
Risk factors:
⁃ Previous uterine surgery, C/S, D&C
⁃ Childbirth
⁃ Middle age
Clinical presentation:
⁃ Commonly report symptoms similar to those reported by patients with (ddx) endometriosis.
⁃ Common complaints include menorrhagia, dysmenorrhea, metrorrhagia, chronic pelvic pain and
dyspareunia
Physical Examination:
- Enlarged and tender uterus!!
38
Another similar recall:
39
33. A trauma case. The trachea was shifted to the right side. There was expanded
lungs and mediastinum. The patient was hypotensive.
What is the most likely dx?
A. Thoracic aortic injury.
B. Tension pneumothorax.
C. Cardiac tamponade.
Answer: A
Speciality: Emergency
Explanation:
RIGHT shift of trachea + Wide mediastinum = Thoracic Aorta rupture.
40
34. trauma with hypotension , x-ray : trachea shifted to the right , expanded lungs
and widened
mediastinum. What is the dx ?
A- Massive haemothorax
B- Pneumothorax
C- Thoracic aorta rupture
D- Spontaneous pneumothorax
Answer: C
Speciality: Emergency
Explanation:
RIGHT shift of trachea + Wide mediastinum = Thoracic Aorta rupture
41
35. 7yrs old with rash on her cheeks and joints pain and anima what the most
specific test regarding her disease activity
A- anti ccp
B- bnp
C- ANA
D- Anti dna ds
Answer: D
Speciality: Medicine
Explanation: ()أم اﻟﻘﺮى
Diagnosis: SLE
42
36. Fungal infection in pregnant lady how to treat?
A. Oral antifungal
B. Topical antifungal
Answer: B
Speciality: ObGyn
Explanation:
I think this is the recall:
43
37. A patient came to ER (long scenario) he has weakness and Paresthesia in left leg
with power of 2/5. Other limbs normal. What is the cause?
A- Migraine
B- Right MCA
C- Right ACA
D- Basilar
Answer: C
Speciality: Neurosurgery
Explanation:
44
38. Pregnant with herpes treated with Acyclovir. Rationale of acyclovir usage?
A. Increase immunity of the mother
B. Decrease placental transmission
C. Decrease herpes simplex virus infection
D. Decrease shedding of the virus and duration
Answer: D
Speciality: ObGyn
Explanation:
45
39. 18 years male came with stap wound injury next to umblicus with small opening
clean and bleeding stopped
Vitally stable with normal lab results
Ct : smal splenic laciration with minemum fluid surrounding it .
What is the best definite management
A) wound closure
B) AB
C) splenictomy
46
D) exploratory labromyomy
Answer: A
Speciality: Surgery
Explanation:
47
40. Old man says he sometimes forgets his friends’ names or celebrities in his
community. His wife is worried he has Alzheimer’s.?
Other recall: old man with tender joints, forgets his friends’ names, his wife is worried ,
what diagnosis ?
A- Alzheimer’s
B- Benign-forgetfulness
Answer: B
Speciality: Medicine
Explanation:
48
41. 4 years old girl brought to the pediatric emergency with vommiting and
lathergy. Parent report decrease wight 1 Kg in last week. She has decrease in apetite.
Thrist and frequent urination. Upon examination she looks sleepy and lathergic. Dry
mucus membran and increase skin turger. The doctor start treating her with IV RL with
electrolyte low dose insulin infusion.
Her PH 7.2 ( not sure ) glucose ( 20 ) +ve urine kitones
What is the suspected complications
A. hypoglycaemia
B. Renal faliure
C. Cerebral edema
Answer: C
Speciality: Pediatrics
Explanation: (UpToDate)
- Cerebral injury accounts for the majority of deaths (60 to 90 percent).
- Cerebral injury generally develops during the first 12 hours of treatment but can
also occur before treatment [35,55]. Throughout the course of treatment for DKA, all
children should be carefully monitored for signs and symptoms that suggest cerebral
injury, which include changes in mental status, urinary incontinence, and new
headache or recurrence of vomiting [9].
49
42. Young male had pharyngitis, then cough and fever, what is the most
likely organism?
a) Staph aureus
b) Streptococcus pneumonia
Speciality: Medicine
Explanation: (UpToDate)
Another recall:
4 months year old boy sore throat decreased oral intake can’t swallow solid food has
enlarged lymph node. vaccinated up to date. What is the most likely organism ?
A- Hib
B- streptococcus pyrogens
C- Staphylococcus arues
D- pneumococcus
50
Answer: B
51
43. DM obese on life modifications his A1c 8 what to add
A-dulaglutide is also GLP
B-DPP-4 inhibitors;
C-GLP-1 receptor agonists
D-SGLT2 inhibitors;
Answer: A is a GLP (Some said the first option is biguanide which is metformin and
that makes sense, and if so, choose A)
Speciality: Medicine
Explanation:
As per أم اﻟﻘﺮى: Metformin is always the first line. If failed, or CI, go for the second line
according to the case which will be dulaglutide (GLP-1 receptor agonist) for sure.
52
44. 36 year-old female presented with left neck mass 2x2cm in posterior angle of
mandible. US: Normal thyroid, left large LN with cystic component. FNA: All smear
shows follicular thyroid What is the most likely diagnosis?
A. Metastatic thyroid cancer
B. Apparent thyroid
C. Ectopic thyroid (in the pathway of thyroid )
D. Thryoglossal cyst
Answer: A
Speciality: Surgery
Explanation:
Similar recall:
33yo male with painless swelling in the posterior triangle of his neck. On US thyroid is
normal and the swelling has cystic component. FNA shows complete follicular cells.
What is the most likely diagnosis?
A- Papillary thyroid cancer
B- Follicular thyroid cancer
C- Ectopic thyroid
D- Thyroglossal cyst
Answer: A
(SMLE Surgery):
This is metastatic thyroid cancer to lymph node.
53
Papillary thyroid carcinoma under microscope is formed of follicular cells.
54
45. Patient with hyperthyroidism and palpitations, heat intolerance.tmt
A. Propriouracil
B. Methimazole????
C. Carbimaxole
D. Propranolol?????
Another recall: case of hyperthyroidism and palpation. What will you do for her as
initial management?
A. PTU
B. Methimazole
C. Propranolol
Answer: D
Speciality: Medicine
Explanation: (BMJ)
Offer a beta-blocker such as propranolol until specific therapy (antithyroid drugs,
radioactive iodine, or surgery) normalises peripheral thyroid hormone levels.[2] Give a
calcium-channel blocker if beta-blockers cannot be used.
Specific therapy: antithyroid drugs, radioactive iodine, or surgery.
Antithyroid drugs, radioactive iodine, and surgery are all effective and relatively safe
options for treating Graves' hyperthyroidism. Discuss the possible benefits and risks of
these treatment options and the likelihood of a good response with patients (and their
parents and carers, as appropriate), and take their preferences and values into
account in addition to their clinical characteristics.
(Amboss):
55
56
46. 11 year old girl with excesize intolerance and wight (did not mention loss or
gain) Hx is unremarkable and PE is normal Pb:160/100 HR:130 T:36.6 O2: 95
ECHO shows normal heart function and anatomy. What is the most appropriate ?
managment?
A. ABG
B. electrolyte level
C. CBC ??? (anemia )
D. TFT????
Answer: D
Speciality: Pediatrics
Explanation:
Presentation goes with hyperthyroidism.
(Amboss):
57
47. unilateral neck swelling in the RT side by investigations : hot thyroid nodule
remaining of the gland cold TSH is low, T3, T4 high No LN enlargement (dx
hyperthyroidism toxic nodule) What is the initial Treatment?
A- antithyroid drug
B- RT thyroidectomy
C- Hemithyroidectomy
D- radioactive iodine
Answer: A
Speciality: Medicine/Surgery
Explanation: (Amboss)
It is toxic adenoma.
- Definitive management is radio-iodine or hemithyroidectomy.
- If the patient is symptomatic> antithyroid first to achieve euthyroid status before
any further management.
(they asked for INITIAL)
58
48. unilateral neck swelling in the RT side by investigations : hot thyroid nodule
remaining of the gland cold TSH is low, T3, T4 high No LN enlargement (dx
hyperthyroidism toxic nodule) What is the initial Treatment? Repeated
A- Antithytoid med
B- Radioactive ablation
C- lobectomy
D- Total thyrodectomy
Answer: A
Speciality: surgery
Explanation:
59
49. Cold thyroid nodule 3x3 was removed by thyroid lobectomy , 8mm papillary will
defined focus was found distant to the leison, what is the appropriate?
A. Complete thyroidectomy
B. Follow up 3 months
C .RAI
Answer: B
Speciality: Surgery
Explanation:
60
50. Middle age Pt with papillary thyroid cancer , planned for total
thyroidectomy how to follow up ?
1- serial post op US
2- calcitonin
3- TSH , T3 , T4
4- Thyroglobulin
Answer: D
Speciality: Surgery
Explanation:
This answered was reached after my reading in Schwartz page 1648 on surgical
treatment part of PTC, if the choices include RAI will be the best. READ ABOUT IT !
61
51. Patient with hyperthyroidism on anti thyroid medications for 10 months, still
symptomatic. TSH: 0.1 and T4 was 27 I think. Best next step?
A. Continue anti thyroid
B. Subtotal thyroidectomy
C. Near total thyroidectomy
D. RAI or increase the dose of drug
Answer: D
Speciality: Surgery
Explanation:
62
52. 28 year old female presented following radiation of neck with neck swelling.
Ultrasound showed swelling in the left lobe of the thyroid of 3x4cm of papillary
thyroid cancer. Which of the following is the best management?
A) Left Lobectomy
B) Left Lobectomy with Isthmusectomy
C) Subtotal Thyroidectomy
D) Total Thyroidectomy
Answer: D
Speciality: Surgery
Explanation:
63
53. 48 yo lady with diffuse goiter, high T4 low TSH, US show bilateral thyroid
nodules , right 3x4 in size , left is 1x2 size what to do?
A. FNA both
B. FNA the larger one
C. total thyroidectomy
Answer: C
Speciality: Surgery
Explanation:
64
54. 2 weeks post thyroidectomy came with high tsh how to manage?
A. reevaluate after 2-3 months
B. Increase or decrease the dose
Answer: A
Speciality: surgery
Explanation:
65
55. Hypothyroidism clear Hx and labs with low Na (it was 129) normal 134
What to do next ?
1. Levothyroxine
2. treat the hyponatremia with hyper Na fluid
Answer: A
Speciality: medicine
Explanation:
66
56. A patient with renal failure presents with low calcium, high alkaline phosphatase,
and high PTH. What is the best form of vitamin D to give this patient?
A- Calcitriol
B- Vitamin D2
Answer: A
Speciality: Medicine
Explanation:
67
57. Most effective method to decrease stone formation in pt with hypercalcemia ?
A. decrease ptn
B. thiazide
C. allopurinol
D. decrease calcium intake
Answer: B
Speciality: Medicine
Explanation:
68
58. Pt with diffuse thyroid enlargement on exam there is 1 nodule in each lobe, labs
showed hyperthyroidism what will you do?
A. Thyroid scan.
B. FNA from both nodule
Answer: A
Speciality: Medicine
Explanation:
69
59. 32 year old pregnant lady in 3rd trimester was complaining of palpitations and
sweating and heat intolerance along being restless she have notice a large goiter but
no dyspnea or difficult swallowing and change in both orbits that became more
prominent. ECG - sinus tachycardia. CBC -LFT - RFT within normal. TSH 0.1 ( low ) , T4
10 ( high ). Thyroid Scan showed diffuse homogenous uptake. Anti-Thyroid
Stimulating Antibody is pending. what is your next step for management ?*
A. Thyroidectomy
B. Radio-active Iodine Therapy
C. Methimazole
D. Propylthiouracil.
Answer: C
Speciality: Medicine
Explanation:
70
60. A 72 year old man presented with an episode of right sided weakness that
lasted 10 minutes and fully resolved and he is clinically stable, he has no other medical
illness. On examination: BP 110/70, HR 95, T 36.6 Which of the following is the most
appropriate next step in the management?
A.Aspirin.
B. Warfarin, INR 3-4
C. Warfarin, INR 2-3
D. No additional drug treatment
Answer: A
Speciality: Medicine
Explanation:
71
61. A 67 year old male is brought to the ED by his wife saying that he developed
weakness and dysarthria after he woke up. She says that he went to sleep having no
complaints, then woke up 2 hours later with symptoms of weakness and slurred
speach. O/E power 2/5 in left side and 5/5 on the right side. CT picture included and
shows hypodensity in the right hemisphere. What is the management?
A. clopidogrel
B. Aspirin
C. Thrombolytic
Answer: C
Speciality: Medicine
Explanation:
It is a stroke within the golden period 4.5h. CT findings exclude (Hemorrhagic stroke).
72
62. 65 year-old male patient k/c of DM, HTN and PAD, presented with severe
claudication, admitted to the hospital as a case of acute limb ischemia and received
thrombolytic therapy. Which of the following is a cause of death during hospital
admission?
A. Bleeding
B. Hemorrhagic stroke
C. Pulmonary embolism
D. Myocardial infarction
Answer: B
Speciality: Medicine
Explanation:
73
63. patient complaining of chest pain, diaphoresis and other symptoms of Ml that
started 2 hours ago. He has a history of ischemic stroke (2 months ago). what would
you give him next? ]
A. Thrombolytic Therapy
B. Aspirin
C. PCI
D. Statin
E.
Answer: B
Speciality: Medicine
Explanation:
Next: Aspirin…… Best: PCI
74
64. 50 years male returned from vacation ,drowsy, poor concentration,difficult
awaking from sleep ,he described multuple fulls but all labs and brain images are free
A. Post concussion syndrome
B. Chronic subdural hemorrhage
Answer: A
Speciality: Medicine
Explanation: (AMBOSS)
A is the best answer because there is sleep disturbance (absent in chronic subdural
hematoma) Also, chronic subdural hematoma would appear hyperdense in CT
(Hyperdense in MRI)
75
65. Child diagnosed of epilepsy on medication, what is true regarding his
vaccincation?
A- Give all.
B- Don’t give live vaccincation.
C- Hold Dtap.
Answer: A (corrected)
Speciality: pedia
Explanation:
76
66. 9-month Hx of MI came to clinic for smoking cessation he was in respiratory
distress raised JVP nicotine tear in his hand what is the most appropriate drug?
A. Nicotine replacement therapy
B. Varenicline
C. Bupropion
Answer: B
Speciality: medicine
Explanation:
77
67. Pt with systemic sclerosis ..comes with fatigue and dizziness Bp188/90 Renal
impairment. What medications to add?
A.Lisinopril
B.Amilodipin
C. BB
Answer: A
Speciality: medicine
Explanation:
78
68. 70 old male with forgetfulness, progressive gradually for 2 years.. devolop signs
of aggression and violence after he was calm ,caring ,and kind:
A. Lewi body
B. Alzheimer’s
C. Vascular dementia
Answer: C (corrected)
Speciality: Medicine
Explanation:
79
69. child had meningitis, and he contacted his brother & sister, what will give as
prophylaxis ?
A. penicillin + rifampicin 2 times
B. penicillin
C. Ciprofloxacin one dose
D. Ceftriaxone 3 doses IM
Answer: C
Speciality: pedia
Explanation:
80
70. 78y/o male presented with change level of consciousness and fever , lumber
puncture was doe show CSF gram +ve bacilli , hem catalase +ve , how will you treat ?
A-ceftriaxone + vancomycin + steroid
B- ceftriaxone + ampicillin + vancomycin
C- ceftriaxone + vancomycin + ampicillin
D- ampicillin
Answer: D
Speciality: MEDICINE
Explanation:
i think it’s D Listeria is the organism and has been confirmed so no need for empirical
Ab Only give ampicillin or in combination with gentamicin
81
71. Question about Meningitis, a 7-year-old patient infected with a gram- positive
diplococcus.Treatment is:
A. Ceftriaxone
B. Penicillin + gentamycin
C. Gentamycin
D. Gentamycin plus vancomycin
Answer: A
Speciality: PEDIA
Explanation:
82
72. In a patient with suspected transient Ischemic attack full recovery should take
place within:
A. 6 Hours
B. 12 Hours
C. 18 Hours
D. 24 Hours
E. 48 Hours
Answer: D
Speciality: MEDICINE
Explanation:
83
73. old patient came with subdural hematoma with signs of lateralization imaging
revealed 13 mm shifting. his GCS 7/15 then was intubated and resuscitated what to
do next
A- iv mannitol
B- elevate head of bed
C-hyperventilate
D- urgent craniotomy
Answer: D
Speciality: SURGERY
Explanation:
Guidelines recommend surgical evacuation for pt with acute SDH who have clot more
than 10mm or midline shift more than 5mm regardless of GCS score
SCHWARTZ
84
74. Pediatric patient had a fall from 1 story high and direct trauma to the head,
presents with hemotympanum No loss of consciousness, no vomiting , neuro exam
Normal Ear : Ruptured tympanic membrane with intact external auditory canal Most
likely bone fracture:
A. Mastoid
B. Maxillary
C. Basal skull
Answer: C
Speciality: SURGERY
Explanation:
85
75. stroke patient developed new weakness on ex papilledema CT finding:
hemorrhagic stroke. Most definitive Mx ?
A. Craniotomy and decompression
B. Iv mannitol
C. iv dexa
Answer: A
Speciality: SURGERY
Explanation:
86
76. A patient who presented with signs of stroke and a CT shows subarachanoid
hemorrhage. He was resuscitated but he is still hypotensive. What is the next best
step?
A. Craniotomy and evacuation
B. Iv mannitol
Answer: A
Speciality: SURGERY
Explanation: THE PATIENT IS HYPOTENSIVE
87
77. Elderly had cough with white sputum that turned yellowish with
diffused infiltration bilaterally and more dense in right lung . Dx?
A. Atypical pneumonia
B. pneumococcal pneumonia
C. Viral pneumonia
88
78. What is the AB that use in ICU patient have pneumonia:
A. Vancomycin
B. Azithromycin
C. Ceftoxime
D. Amoxicillin
Answer: A (corrected)
Speciality: MEDICINE
Explanation:
89
79. What causes interstitial pneumonia / Interstitial pneumonitis?
A-Viruses
B- tuberculosis
C- bacteria (Atypical pneumonia)
Answer:
Speciality:
Explanation:
90
80. A case of interstitial pneumonitis most commonly associated with?
A-Viral pneumonia
B- Lobar pneumonia
C- Bronchopneumonia
D-Secondary tuberculosis
Answer: A
VIRAL OR ATYPICAL
Speciality: MEDICINE
Explanation:
91
81. infant or child!! with pneumonia and atelectasis, what is best
Investigation:
a) fluoroscopy
b) rigid bronchoscopy
c) flexible bronchoscopy
d) x-ray
Answer: D
Speciality: Pedia
Explanation: pneumonia is a radiological diagnosis
92
82. After major operation for resection for ischemic gut , patient developed
fever 38 , labs normal except for TLC 11 Why?:
A. Pulmonary embolism
B. Inflammatory reaction
C. Pneumonia
Answer: B
Speciality: Surgery
Explanation: Fever in the surgical patient - UpToDate
93
83. Old man after episode of cough has SOB and abdominal pain on X- ray there is
air fluid level in the neck and under the diaphragm and left pleural effusion . BP: 90/65
HR : 115/ min RR : 25 . SO2: 90 on room air what is the most important mangment ]
A- Bulla of lung
B- Laparotomy (perforated viscus )
C- Chest tube drainage-
D- Observation
Answer: B
Speciality: Surgery
Explanation: I am not sure about the air in the neck, but here is Earth’s answer
94
84. 80 year old smoker for 40 years , all labs normal except high Ca , Dx ?
A. paget dis
B. hyperparathyroidism
C. lung cancer
Answer: C
Speciality: Medicine
Explanation:
Seriously short question with limited information, however I would go with cancer for
the age and smoking, but I think there will be more information so do not skip the
part below:
- Not A for sure because it presents with isolated increased ALP levels and normal
calcium level.
- Not B because it is not related to smoking, and parathyroid cancer is extremely
rare. Also, it would have other features mentioned below.
95
85. 32 non smoker presented with lung nodule 7 mm otherwise normal except for
ca 4 mmol :
A. Repeat CT scan within 3-6 months
B.Pulmonary funtion test
C. Biopsy of the lesion
D. PET scan
Answer: D
Specialty: Medicine
Explanation:
Patient has hypercalcemia
96
86. Copd patient his FEV1% is 39 he is on inhaled salbutamole (SABA) and
ipratropiam and still not controlled What to add ?
A. Inhaled salmeterol
B. Inhaled slameterol + Inhaled corticosteroid Long o2 therapy
Answer: B
Speciality: Medicine
Explanation:
97
87. COPD pt on high flow o2 3 L with O2 saturation 93% , develop acidosis and
hypercapnia, what will you do ? ]
A. Decrease O2 flow
B. Mechanical ventilation
C. Increase O2 flow
D. CPAP
Answer: A
Speciality: Medicine
Explanation:
98
88. Preschool anemia screening (with no symptoms only this statement) CBC: Low
Hg High Retcs High RDW (>2%) what is dx: ]
A. IDA
B. Thalasemia minor
Answer: B
Speciality: Medicine/Pedia
Explanation:
Limited information, but it is B according to amboss
99
89. Chronic kidney disease picture of iron def anemia and and high ferritin what is
the treatment ?
A Oral iron
B erythropoietin
C IV iron
Answer: B
Speciality: Medicine
Explanation: I really could not find a reliable source for this information, but for sure
you will not give iron to a patient with elevated iron level ()ﺑﯿﻄﻔﺢ ﺣﺪﯾﺪ
However, most CKD patient present with low iron level, and in that case, oral iron
supplements would be the answer unless IV is indicated (e.g. celiac disease)
100
90. Child pt with decreased IQ, pica (eat dirt) < they wrote it :) With
Hepatosplenomegaly ! Labs showed *Hypochromic microcytic anemias.
What is the highest diagnostic labs ]
A- blood lead levels
B- CBC with blood smear (they already do it )
C- Iron studies
Answer: A
Speciality: Pedia
Explanation:
I believe this the same recall of lead poisoning (low socioeconomic status, low IQ,
pica), therefore we will go with lead level in the blood.
Lead poisoning can cause cognitive impairment (hence the low IQ)
101
91. Sickle cell trait, what’s the most common complication during pregnancy?
A- IUGR
B- Chest infection
C- Preeclampsia
D- UTI
Answer: D
Speciality: Ob/Gyn
Explanation:
I searched in ACOG and got their source regarding sickle cell trait; it does NOT
mention any of the above complications, thus patients with sickle cell trait will have
normal pregnancy as others. However, pay attention that this is sickle cell trait not
disease. In case it was sickle cell disease, the answer would be IUGR or low birth
weight
102
92. Plt were low, PT&PTT&INR all normal, high bilirubin and reticulocytes and low
Hb (indicates hemolysis), creatinine was normal, your management:
A.Steroids and IVIG
B. Exchanged transfusion
C. Platelet transfusion
Answer: B
Speciality: Medicine
Explanation:
This is mostly a case of TTP, though it usually presents with renal impairment (inc.
creatinine and BUN)
Not A >> Steroids is used for both ITP and TTP, however IVIG is used for ITP but not
for TTP
Not C >> Platelets transfusion is not usually given unless indicated for a procedure
103
93. 81 yo woman DM, HTN on medications at risk or known to have DVT asking
about prophylactic for DVT?
A. Aspirin 81 mg oral
B. Enoxaparin 40mg SC.
Answer: B
Speciality: Medicine
Explanation:
Another recall
A 73 year old woman is brought to ER after a fall at home. She is diagnosed with left
hip fracture (see lab results) Weight 82 kg Sodium 136 (normal) Potassium 4.2 (normal)
Creatinine 68 (normal) What is the best order by the admitting orthopedic surgeon to
prevent deep vein thrombosis?
A- Aspirin 81 mg PO daily
B- Enoxaparin 40 mg SC daily
C- Fondaparinux 10 mg SC daily
D- Heparin sulfate 10,000 units IV BID
Answer: B
Speciality: Surgery
Explanation: UTD
104
94. Female c/o urine leakage with cough, sneezing, exercise and with urgency.
What type of incontinence does she have?
A. Urge incontinence
B. Stress incontinence
C. Mixed incontinence
D. Overflow incontinence
Answer: C (corrected)
Speciality: medicine
Explanation:
105
95. 80 yr old male c/o inability to urinate with Abd. Pain and severe need of
urination What to do?
A. Cystoscopy
B. Emergency TURP
C. Foley cath + culture
Answer: C
Speciality: Medicine
Explanation: Next step is bladder decompression, best is TURP as it decrease the
retention by 85-90%
106
96. Screening for bacteriuria in pregnancy ?
A. 12 weeks
B. 25 weeks
C. 30 weeks
Answer: A
Speciality: Ob/Gyn
Explanation: 12 - 16 weeks.
107
97. Female came to ER with sign and symptoms of appendicitis, appendectomy was
done , surgeon discover the appendix was normal , what surgeon should be do ?
A. Tell pt what happened and that was routine and no complication
B. Not tell the pt
C. Write report about what happened and give it to hospital committee
Answer: A
Speciality: Surgery
Explanation: SMLE Brain
108
98. Patient old with end stages cancer disease complaining of severe pain and his
children ask you to give him stronger pain killers you respect theirs wishes but you
told them that the pain killers may fasten the death of their father question in this case
what the doctor do?
A- principles of double effects
Answer: A
Speciality: Ethics (Misc)
Explanation:
109
99. You want to do surgery to a 15 y old , from where take the consent ;
A. Parents alone
B. Patient alone
C. Both of them
D. Patient’s assent and parents’ consent
Answer: D (Corrected)
Speciality: ethics
Explanation:
110
100. couples with infertility ask about surrogate pregnancy which not allowed in ur
hospital and u are not sure wither allowed in saudi or not?
A. Warn them might not be allowed in ksa.
B. appointment to check yourself and then tell them.
C. Referral to colleague who can help to do it.
Answer: B
Speciality: ethics
Explanation:
111
101. Female patients come to the doctor's office with signs of abuse. She told you
the abuse was from her husband and told you not to tell her husband. What should
you do?
A. Threatens the husband by police
B. Tell the police
C. Do what the patient said to you
Answer: B
Speciality: ethics
Explanation:
112
102. A patient underwent C/S and developed massive bleeding that was not being
controlled by medical management. A decision was made by the consultant but the
consent was not obtained, what is the most appropriate action?
A - obtain consent from husband
B - perform hysterectomy without consent
C - consult the ethics committee
D - do not perform the hysterectomy
Answer: B
Speciality: ethics
Explanation:
113
103. A new drug is approved by the FDA and the doctor is a shareholder in the
company
A. conflict of interest
Answer: A
Speciality: ethics
Explanation:
114
104. lactating mother unilatral nipple inversion. US : duct dilation 3.5 cm. What is the
Dx :
A-Breast cyst
B-Breast cancer
C-Duct ectasia
D- breast abscess
Answer: C
Speciality: OBGYN
Explanation:
115
116
105. known case of liver cirrhosis secondary to Hepatitis C has completed treatment.
Hepatitis C RNA is negative. How will you follow up this patient?
a. Regular screening with AFP
b. Regular screening with ultrasound
c. Liver biopsy
d. No follow up is required
Answer: B
Speciality: medicine
Explanation: Um alqura
117
106. Pregnant came with candida like picture and then asked what’s the TTT ?
A-oral metronidazole
B-azithromycin
C-topical metronidazole
D- topical fluconazole
Answer: D
Speciality: OBGYN
Explanation:
118
107. 4 y.o his went to doctor because parents recognize blood in stool this the only
symptoms Then the doctor examined him and said it’s common and can be
preventable:
A- anal fissure
B- hemorrhoids
C- juvenile poly
D- Intusseption
Answer: A
Speciality: pedia
Explanation:
119
108. 3yr old brought to ER after RTA diagnosed with massive intra-abdominal
bleeding and severe splenic injury underwent a exploratory laparotomy and
splenectomy. Which vaccine to give?
A. Meningococcal
B. OPV
C. VARICELLA
Answer: A
Speciality: surgery
Explanation: same idea, after splenectomy we give meningococcal vaccine
120
109. 25 weeks pregnent ,sero-ve for rubella , when take vaccicine?
A- now
B- postpartum
C- no need
D- 3d trimester
Answer: B
Speciality: OBGYN
Explanation:
121
110. Pediatric case h/F sore throat & fever then 3 days he develops body rash on the
thighs and buttocks.Lab finding suggest renal insult with anemia and normal Plat
count) plus ll edema +black tea urine.
A- post streptococcal GN
B- HUS
C- HSP (Henoch-Schonlein Purpura)
Answer: C
Speciality: pedia
Explanation:
122
111. Typical symptoms of croup, how to manage?
A. Dexa and epinephrine
Answer: A
Speciality:
Explanation:
123
112. Baby term delivery (didn't specify CS or SVD). 4 hours later complaining from
SOB , Tachypnea, Tachycardia with grunting
A- Hyaline membrane disease preterm
B- pulmonary "something"
C- Meconium Aspiration syndrome
D-Transient Tachypnea of the New born
Answer: D
Speciality: pedia
Explanation:
124
113. A long scenario of a pediatric with open fracture. You see his thigh covered with
gauze full of blood. Hypotensive but awake and airway potent. What is the best next
thing to do?
A. Call surgery on call
B. Give bolus NS
C. Give morphine
D. External fixation
Answer: B
Speciality: surgery
Explanation: ABC protocol, airway is potent then go for circulation
125
114. Pregnant in third trimester presented with bleeding, cervix was 7cm dilated.
Whats the type of bleeding?
A- post partum
B- antepartum
C- intrapartum
Answer: C
Speciality: OBGYN
Explanation:
126
115. husband has 2 siblings with SCD the wife not mentioned , what is the likelihood
of affecting his children to be diagnosed with SCD?
A. Very high
B. Not at risk
C. Low risk if mother negative
D. Advice not to have children
Answer: C
Speciality: Medicine
Explanation: the children will be carrier if the mother is negative because inheritance
of SCD is autosomal recessive
127
116. decrsndo diastolic murmur?
A. Pr
B. Ms
C. Ar
Answer: C
Speciality: Medicine
Explanation: Auscultation of cardiac murmurs in adults - UpToDate
128
117. A patient with pulmonary embolism that wants contraception. WHAT is the most
appropriate contraceptive method?
A) condom
B) IUD
C) OCP
D) POP
Answer: B (corrected)
Speciality: OBGYNE
Explanation: Dr wafa
129
118. Old age pt with cardiomegaly. he's asymptomatic Echo showed: EF 40%. what is
the best Mx
A- digoxin
B- Lisinopril
C- echo after 6 months
D- frusemide
Answer: B (corrected)
Speciality: Medicine
Explanation: Congestive heart failure - AMBOSS
Um alqura
130
119. A 67 year old male presents with insomnia, irritability and palpitations for 3
months. He is known to have HTN, depression and Atrial fibrillation. He is on
amiodarone, fluoxetine, and enalapril. Although he complains of palpitations, he has
no increased heart rate. Clinically, he is unremarkable. Vitally stable. What is the most
appropriate next step?
A. Add propranolol
B. Do thyroid function tests
C. Substitute antidepressant drug
D. Refer to psych
Answer: B
Speciality: Medicine
Explanation: Amiodarone - AMBOSS
131
120. A diabetic lady presenting to the clinic asking about travelers diarrhea
prophylaxis. Mild Cr Elevation, BUN is high, urea is high. What prophylaxis to give?
A.Fluoroquinolones
B. No need
C. Probiotics
D. Bismuth
Answer: A
Speciality: Medicine
Explanation:
132
121. CTG w/ prrolongrd decelation. Most appropriate to do?
A) emergency CS v
B) hydration, reposition and assess in 20 mints
Answer: B
Speciality: OBGYNE
Explanation: Dr Wafa (page 110)
133
122. Male k/c of acute pancreatitis came with abdominal pain in RUQ In US :
gallbladder stone and dilated CBD 1.4 What is your management
A- laparoscopic cholecystectomy
B- ERCP
C- ABx
D- MRCP
Answer: B
Speciality: Surgery
Explanation: Choledocholithiasis - AMBOSS
134
123. Pt after fell down from hight present with open fracture and dirty , what the most
appropriate next step in the management ?
A- oral antibiotic
B- open fixation
C- surgical debridement
D- external fixation
Answer: C
Speciality: Surgery
Explanation: General principles of acute fracture management - UpToDate
135
124. Pt known case of dm , discharged after inguinal hernia repair, what it is your
advise to him ?
A) Abdominal plinder
B ) avoid lifting heavy objects for 6 months
C) laxitive or frusimode i cant remember
Answer: B
Speciality: Surgery
Explanation: Classification, clinical features, and diagnosis of inguinal and femoral
hernias in adults - UpToDate
136
125. Which one of the following indicates Rheumatoid arthritis in pleural effusions?
A- ph >7.6
B- Glucose <1mmol
C- Transudative
Answer: B
Speciality: Medicine
Explanation: Overview of pleuropulmonary diseases associated with rheumatoid
arthritis - UpToDate
137
126. which of the following is more specific for SLE
A.Anti-dsDNA
B.Anti-Smith
C. AntiCCP
Answer: B
Speciality: Medicine
Explanation:
138
127. An 81 year old male presenting with memory loss and difficulty remembering
grandsons name, medically free apart from occasional alcohol intake. What is the
likely diagnosis of his presentation?
A. MultiInfract Dementia
B. Alzahimers Disease
C. Alcohol Induced encephalopathy
D. Parkinsonism with Dementia
Answer: B
Speciality: Medicine
Explanation: Alzheimer disease - AMBOSS
139
128. Patient present with “ring in his eyes” what is most likely diagnosis
A. Wilson's Disease
B. Hemochromatosis
Answer: A
Speciality: Medicine
Explanation: Wilson disease - AMBOSS
140
129. Child with Supracondylar fracture, he has absent pulse, Next step?
A, K-wire Fixation
B, Explore
Answer: B (corrected)
Speciality: Surgery
Explanation: Evaluation and management of supracondylar fractures in children -
UpToDate
If there is closed reduction, it would be a better answer
141
130. Child sits in tripod position, reaches out to things, flips from prone to supine and
vice versa, cannot hold cereal properly between two fingers how old is he?
A- 3 months
B- 6 months
C- 9 months
Answer: B
Speciality: Pedia
Explanation: Dr Safdar
142
131. Ectopic pregnancy case initial BhCG 2900, given methotrexate, one week later
BhCG 6000 what to do:
A- repeat methotrexate
B- Diagnostic laparoscopy
Answer: B (corrected)
Speciality: OBGYNE
Explanation: Dr Wafa
143
132. Long case with Kayser–Fleischer ring and low Ceruloplasmin how to treat?
A- Penicilliamine
B- EDTA
Answer: A
Speciality: Medicine
Explanation: Wilson disease
144
133. Child was in a farm then presented with signs organophosphate poisoning
(didn’t mention dx) how to treat?
A) Atropine
B) Naloxone
C) Flumazenil
D) Sodium Bicarbonate
Answer: A
Speciality: Pedia
Explanation:
145
134. Man come to the ER for poly Truman and he was intubated after resuscitation
admitted to the ICU what is the best approach to exclude cervical spine injury and
remove neck collar?
A. MRI neck
B. Ap and lateral x-ray neck
C. Clinical judgment
D. Ct neck
Answer: D
146
Speciality: Surgery
Explanation:
147
135. Old patient with altered lvl of consiousness fell from the stairs, was brought to
ER, done CT for him showing epidural hematoma. Asking about most likely affected
artery:
A. Pontine
B. Basilar
C. Anterior cerebral artery
D. Middle meningeal artery
Answer: D
Speciality: Surgery
Explanation:
148
136. 36 Year old male patient diagnosed with WPW and he is on BB but still
uncontrolled. What to do?
A- Add CCB
B- Increase the dose.
C- Give amiodarone.
D- Radiofrequency ablation
Answer: D
Speciality: Medicine
Explanation:
149
137. pediatric with seizure and other signs, labs showed metabolic acidosis, urine
analysis: aminoaciduria and glycosuria, and cataract what’s the diagnosis?
A. Galactosemia
B. Homocystinuria
C. Methylmalonic Academia
D. Maple syrup urine syndrome
Answer: A
Speciality: Medicine
Explanation: Amboss + UpToDate
150
151
138. G3p0, A2 now at 5 weeks presented with spotting on examination open os and
no active bleed. History showed 2 abortions at 2nd trimester, last one with D&C
diagnosed as incompetent cervix. Your diagnosis now for the third pregnancy of this
patient?
A- Asherman syndrome
B- Incompetent cervix
C- Chromosomal anomaly
Answer: C
Speciality: ob/gyn
Explanation: 3 different questions, do not get confused.
152
139. Patient with thyroid mass measuring 2*3 on one lobe, patient is asymptomatic,
TSH and T4 are normal, Bethesda 4, what’s next step in management?
A. Hemithyroidectomy
B. Nodule excision
C. Total thyroidectomy
Answer: A
Speciality: Surgery
Explanation:
153
140. A patient has had an oil dissolvant injected by a mistake in his fingers what is the
appropriate management?
1. Surgical removal of the dissolvent
2. Put ice and left it
3. Antibiotic
4. Steroids
Another recall with same options: A mechanic had an accidental injection of oil
dissolving in his index finger Finger is red and swollen with mild tenderness and
normal joint movement what to give:
Answer: 1
Speciality: Surgery
Explanation:
154
141. Stepping reflex when disappear
A. 2 months
B. 4 months
C. 6 months
D. 8 months
Answer: A
Speciality: PEDIA
Explanation:
155
142. Patient suspected to have asthma , which of the following is the most definitive
way to diagnose asthma ?
A- History , Examination, response to bronchodilators
B- Spirometry and PFT
Another recall: Pediatric 5 yrs suspected to have asthma , which of the following is
the most definitive way to diagnose asthma ?
A- History , Examination, response to bronchodilators
B- Spirometry and PFT
Answer: B
Speciality: Pedia
Explanation: If < 5y, spirometry cannot be performed, a trial of medications may help
to establish reversibility.
156
157
143. Soldier going to southern region of Saudi Arabia. What is the best malaria
prophylaxis?
A. Malarone
B. Atovaquone-proguanil
C. Mefloquine
D. Chloroquine
Answer: B
Speciality: Medicine
Explanation: CDC
https://www.cdc.gov/malaria/travelers/country_table/s.html
158
144. Case of molar pregnancy (very high Bhcg...), what is the highest diagnostic
test??
A. US
B. Direct biopsy
Answer: B
Speciality: Ob/Gyn
Explanation:
159
145. A 73 year old woman is brought to ER after a fall at home. She is diagnosed with
left hip fracture (see lab results) Weight 82 kg Sodium 136 (normal) Potassium 4.2
(normal) Creatinine 68 (normal) What is the best order by the admitting orthopedic
surgeon to prevent deep vein thrombosis?
A. Aspirin 81 mg PO daily
B. Enoxaparin 40 mg SC daily
C. Fondaparinux 10 mg SC daily
D. Heparin sulfate 10,000 units IV BID
Answer: B
Speciality: Medicine
Explanation: BMJ
160
161
146. Term Female came for labour , her prenatal history was urine culture (not vaginal
swab for culture) positive 100000 GBS and asthma wich is managed by SABA , now
what to give during labor and delivery?
A. Ampicillin
B. betamethasone
Answer: A
Speciality: Ob/Gyn
Explanation:
162
147. Pelvic Fracture with injury to membranous part of urethra. Blood seen in external
meatus. What is the most appropriate action?
A. Cystoscopy
B. Folly’s Catheter
C. Suprapubic catheter
D. CT pelvis
Answer: C
Speciality: Surgery
Explanation:
163
148. Case for sudden pain in the left leg associated with reduce sensation and color
change?
A-US
B- Ct angio
C- conventional radio
164
149. Pt complain of profuse discharge and tender vaginal fornix
A. Acute salpingitis
B. Acute cervicitis
C. Appendicitis
Answer: A
Speciality: OB GYN
Explanation:
165
150. Case of pancreatic cyst for 5 Ws with collection was 18 cm x 24 cm how to
manage?
A- Percutaneous drainage
B- Endoscopic drainage
C- Surgical drainage
Answer: B
Speciality: Surgery
Explanation:
For Pancreatic pseudocyst, the role is as the following;
- Less than 6 cm & 6 weeks -> Only observation.
- More than 6 cm & 6 weeks -> Drainage
Endoscopic vs percutaneous?
- If there’s signs of infections like high WBC, fever ect.. (regardless the duration or
size) -> percutaneous
- Otherwise, do endoscopic.
In another way:
If infective -> Do percutaneous drainage
If non infective (depend on the size and duration)
- Less than 6 cm & 6 weeks -> Only observation.
- More than 6 cm & 6 weeks -> Endoscopic drainage
166
151. Patient diagnosed with Ulcerative Colitis 6 years ago. when to be screened for
colon cancer?
A. Annually till age of 50
B. If there is mild inflammation
C. At time of diagnosis
D. 8-10 years after diagnosis
Answer: D
Speciality: Medicine
Explanation:
167
152. Female, G1P0, missed her period for 3 months, has irregular period, doesn’t
know gestational age. On Ultrasound, 11 weeks was calculated. What is the most
likely mthod of dating gestational age here?
A- crown-rump length
B- femur length
C- estrogen or progesterone Lvl
D- abdominal circumference
Answer: A
Speciality: OB GYN
Explanation:
168
153. 2 month old baby at the clinic what to do?
A. BMI
B. Head circumference
Answer: B
Speciality: Pediatric
Explanation:
In pediatric population BMI is measured from 2 years upward.
169
154. Newborn/infant with stridor worse when prone improves when supine, mother is
concerned, what to do?
A Start laser treatment
B Tell her it will resolve on its own by first birthday
C Tell her it will get worse
D-Do Laryngoscope
Answer:
Speciality: Pediatric
Explanation:
170
155. 14 y female complain of vaginal bleeding in interval of 3 weeks to 2 months, she
has normal development and normal secondary features, every thing normal What to
do?
A- Reassure
B- OCP
C- Order FSH and prolactin
Answer: A
Speciality: Ob Gyne
Explanation:
Because of her age.
171
156. Pt who is allergic to penicillin which of the following is contraindicated
A-Ceftriaxone
B-Tazocin
Answer: B
Speciality: Medicine
Explanation:
172
157. Patient presented to the ER with diarrhea, nausea, vomiting, salivation,
lacrimation and abdominal cramps. What do you suspect?
A) Organophosphate poisoning
B)Paracetamol poisoning
C) Aspirin
D)Penicillin
Answer: A
Speciality: Medicine
Explanation:
173
158. Known SCA and received blood transfusion 3 weeks ago Now he came for his
hepatitis A vaccine You should :
A-Give the vaccine
B-ask him to come after 3 months
C-Ask him to come after 6 months
D-Ask him to come after 9 months
Answer: A
Speciality: Medicine
Explanation:
Blood transfusion is not a contraindication of vaccination.
174
159. Patient smokes 20 cigarettes and vital signs show BP 140/92 Which of the
following is the most considerable risk factor for MI ?
A) Hypertension
B) Smoking
C) Age
Answer: B
Speciality: Medicine
Explanation:
Another recall
175
160. A boy with ambiguous genitalia, diagnosed with 17β-Hydroxysteroid
dehydrogenase. What type of trait in this disease?
A) Multifactorial
B) X-linked dominant
C) Autosomal recessive
D) Autosomal dominant
Answer: C
Speciality: Pediatric
Explanation:
176
161. Pediatric patient presented with generalized seizure, hypoglycemia, ketones in
the urine with characteristic odor. What is the dx?
A. Galactosemia.
B. Phenylketonuria.
C. Maple syrup urine disease.
D-Fatty acid oxidation defect
Answer: C
Speciality: Pediatric
Explanation:
Galactosemia comes with metabolic acidosis + glucose in urine + aciduria
Maple syrup urine disease comes with smelly urine.
177
162. Lab technician give a result to nurse through the phone, said 2 then pause after
that proceed 3 5 and the nurse wrote 2.35 but the correct one is 235, the patient had
irreversible liver damage by this mistake, Cause of this condition ?
A- Doctor did not check
B- Lack of communication
C- Technician didn’t check if the nurse got the number correctly
D- Mishandling from the nurse
Answer: B
Speciality: Ethics
Explanation: Ethics file
178
163. A 13 year old boy medically free presented with neck swelling. One of the
relatives recently diagnosed with lymphoma and the family is worried about their
child. Which of the following is an indication for lymph node biopsy?
A- Absence of fever
B- Presence of Hepatosplenomegaly
C- Lymph node size > 1cm
D- Tender and hot lymph node
Answer: B
Speciality: Medicine
Explanation:
179
164. 24 years female diagnose with endometriosis she took NSAID but no benefits.
What is Most appointment mx?
A- ocp
B- laparoscopic excision and ablation
C- Hytsterectomy.
Answer: A
Speciality: Ob/Gyn
Explanation: Amboss
180
165. 8 year old female with Heliotrope Rash and Nodules in Phalanges. Diagnosis?
A) Juvenile Dermatomyositis
B) Oligoarthritic JIA
C) Systemic JIA
Answer: A
Speciality: Pedia
Explanation: Dr. Safder
181
166. 8 years old boy. Mother complained of poor training to bathroom which muscles
is targeted in therapy?
A -Perianal
B -Pelvic floor
C -Rectus muscle
D -Detrusor
Answer: D
Speciality: PEDIA
Explanation: AMBOSS
182
167. which age can we give peanuts and eggs to babies to prevent the risk of getting
allergic?
A. 10 months
B. 14 months
C. 18 months
D. 24 months
Answer: A
Speciality: Pedia
Explanation: Dr. Safder
183
168. Child with history of three infection this year, presented now with pneumonia,
his older brother died and had similar condition, his sisters 3 and 5 years are normal
and well, his work up is significant for low B cell and normal T cell.
A. X-linked agammaglobulinemia
B. Ataxia telangiectasis
C. Chronic granulomatatosis disease
D. Hyper IgE syndrome
Answer: A
Speciality: Pedia
Explanation: Dr. Safder
184
169. pt from India with sign and symptoms of Meningitis , labs wbc : lymphocytes ,
high protein ..
A- TB meningitis
B-viral meningitis
C-Bacterial meningitis
Answer: A
Speciality: Medicine
Explanation: Um-alquraa
185
170. A pregnant woman presented with massive vaginal bleeding from the abruption
placenta and her Hgb: 8.6, BP 84\40, HR140. What is the best management to save
her life?
A- Admit to ICU
B- Immediate Transfusion of 2 packs FFP
C- Call multidisciplinary and rapid response team (RR)
D- Immediate Delivery
Answer: C
Speciality: Ob/Gyn
Explanation: Dr Wafa
186
171. Elderly man presented to the ER with rigid distended abdomen prepared for
laparotomy with hypotension and fever (38) X ray showed free air under diaphragm
What is the best initial resuscitation option ?
A- intubation and ionotropes
B- Broad spectrum Abx
C- Colloid via central line
D- Crystalloid fluid via peripheral line
Answer: D
Speciality: Surgery
Explanation:
187
172. Sandifer syndrome infant case ask about risk incidence in :
A. Post- pertussis infection
B. low birth wight
Answer: B
Speciality: Pedia
Explanation:
Nonepileptic paroxysmal disorders in infancy - UpToDate
188
173. 6 YO known case of congestive heart failure , present in ER with SOB after
playing , HR : 200 other examination normal , ask about apportiate mangment :
A⁃ iv fursamide
B ⁃ observation as symptom expected of his diagnos .
Answer: A
Speciality: Pedia
Explanation: Heart failure in children: Management - UpToDate
189
174. infant with esophagitis diagnosis written in Q , ask about treatment :
A. PPI oral
B. fundplication surgery
Answer: A
Speciality: Pediatrics
Explanation:
Medication is the 1st line of treatment.
190
175. post MVA with urethral injury , what next : (i’m sure no suprapupic tube in
choices)
A. cystoscopy
B. folly catheter
C. retrograde urthrogram
D. fluoroscopic cystogram
Answer: C
Speciality: Surgery
Explanation:
191
176. Petient came with sore throat and...... With purlunt with mebranous.
A. Influnza
B. corona virus
C. RSV
D. EBv
193
This might be the recall:
30 year women complained of dry cough very sever comes in attack that she can not
complete talking intial dx as alergic rhinitis took intranasal steroid but no improvemt
..they mentioned that she has allergy to dust .on exam slight nasal congesion
FEV1/FVC <70 and improved by 20% with bronchodilator ..what you want to do:
A- Chest ct
B- Skin allergy test
C- IgE antibody level
D- Methacholline chalenge test
Answer: B
(MD1TALK SMLE IM) channel):
- CT → no features of churg strauss, or GPA or any other ILD ❌
- IgE → indicated to measure it at moderate-to-severe persistent asthma when
considering treatment with anti-IgE monoclonal antibody (omalizumab) or when
allergic bronchopulmonary aspergillosis is suspected on the basis of eosinophilia ❌
- Methacholine → no need, reversibility of airways is documented by 20% (Fev1
>15%)❌
- Skin allergies test → although it don’t help in diagnosis, its important in
management, helping the patient to avoid his triggers (the answer ✅)
194
178. Man come to the ER for poly Trauma and he has severe back and leg pain you
suspected spinal cord injury , vitaly stable Initial management
A- x-ray
B- Wait neurosurgery consultation
C- Ct
D- Observation
Answer: A
Speciality: Emergency
Explanation: (UpToDate)
195
179. old pt have painless heamaturia whats is most appropriate investigation
A- cystoscopy
B- us
C- ct
Answer: A
Speciality: Urology
Explanation: (Amboss)
196
180. Pt have white coated tounge that removed with scratch and treated with nystatn
what is diagnosis
A- erythymetus candidiasis
B- Leukoplakia
C- oral thrush
Answer: C
Speciality: Medicine
Explanation: (UpToDate)
197
181. 31 year female have pap smear negative in the last 5 year now also clear when
to do it
A- after 1 year
B- after 3 year
C- after 5 year
D- no need
Answer: B
Speciality: ObGyn
Explanation:
198
182. Female pt have history of previous myomectomy and several induction for
fertility due to polycystic ovary
Now present with uterine bleeding what's the cause
A- endometrial hyperplasia
B- endometriosis
C- adenomyosis
D- Uterine fibroid
199
200
183. Pt have hx of trauma (MVA. and develope SOB with decrease air entry in the
right side xray emphysema+ peumomediastinum
Cause?
A- open pneumothorax
B- tension pneumothorax
C- tracheobronchial injury
Answer: C
Speciality: Surgery
Explanation: (UpToDate)
201
184. Pt have unilateral neck pain that increase and become like electeric shock from
neck radiate to shoulder also develop weakness in the arm
Whats cause?
A- cervical disc prolapse
B- polymyalgia rheumatica
Answer: A
Speciality: Neurology
Explanation:
Similar recall:
32 years old male was complaining of neck pain radiate to the right shoulder with
strike electric sensation in the arm
What it the most likely cause?
A- Whiplash injury
B- Cervical disc prolapse
C- Rotator cuff tendonitis
D- Polymyalgia rheumatica
Answer: B
202
185. When to screen pregnant lady for asymptomatic bacteruria?
A- at 12th week
B- 20
C- 26
D- 36
Answer: A
Speciality: ObGyn
Explanation:
203
186. Pt came with anal pain + hx of hemorrhodectoy before 2 month
On exam doctors can't able to do P R due to severe pain what's cause ?
A- anal fissure
B- anal stenosis
C- heamarrhoid
D- anal sphincter
Answer: A
Speciality: Surgery
Explanation: Anal fissures - AMBOSS
204
187. GCS calculation,Eye open to sound. Localise pain..confuse
A- Mild
B- moderate
C- severe
Answer: B
Speciality: Surgery
Explanation:
- Eye open to sound = 3
- Localise pain = 5
- Confuse = 4
3 + 5 + 4 = 12 which is moderate.
205
188. Female Patient on icu urosepsis and septic shock, did not respond to ressisatuve
mesures, he was started on norepinephrine but no response. Check labs below
Right atrial pressure: Normal
PCWP: Slightly high
CO: Normal
SVR: Normal
What is the cause of her deterioration
A-Adult respiratory distress syndrome
B-Volume overload
C-Cardiogenic edema
Answer: A
Speciality: Medicine
Explanation:
206
207
208
189. A female patient presented with joint stiffness, she also has photosensitivity,
malar rash with healed ulcers, her proximal muscle strength is 3/5. Labs
ANA +ve
Rheumatoid factor +ve
CBC shows anemia
Which of the following serves the highest diagnostic value?
A-Anti DsDNA
B-Anti CCP
C-Anti RNP
Answer: A
Speciality: Medicine
Explanation: if there is Anti-smith I will choose it
209
190. Patient presents with fever upon examination there was painful nodules and
splinter hemorrhages. Which of the following is the most appropriate management?
A-Gentamycin
B-Vancomycin + Ceftriaxone
Answer: B
Speciality: Medicine
Explanation: Dx: Infective endocarditis
210
191. Uncontrolled asthma patient present to the ER with asthma exacerbation, his last
many reading of PEF was <450. Which of the following indicates severe asthma
exacerbation?
A-Respiratory rate >25
B-PEF <350
C-Heart rate > 100 bpm
Answer: B
Speciality: Medicine
Explanation:
SINA
Acute severe asthma ·
Any one of the following:
o PEF 30 – 50% best or predicted reading
o Respiratory rate ≥25/min
o Heart rate ³120/min · Inability to complete sentences in one breath
211
192. A 65-year-old patient with yellowish sputum and hemoptysis, he’s a smoker for
30 years.
Vitals: Fever 38
X-ray: Right lower lobe infiltrates
What is the diagnosis?
A-Acute bronchitis
B-Community acquired pneumonia
C-COPD
Answer: B
Speciality: Medicine
Explanation:
(Purulent yellow productive cough, fever, “several infectious manifestations”, high
WBC, x-ray findings) go with Pneumonia more than COPD.
212
193. Primigravida 32W with type 1 DM presented with abdominal pain and uterine
contractions her os is closed what to do?
A-Steroid and increase insulin
B-Steroid
C-Steroid, increase insulin and tocolytics
Answer: A
Speciality: OBGYN
Explanation:
213
194. 9-month baby breast fed and mom started to introduce solid food but he’s
mainly on breast feeding
What is the most appropriate advice?
A-Iron
B-Vitamin D
C-Speed up the process of solid food introduction
D- Nursing bottle
Answer: A
Speciality: Pedia
Explanation:
214
195. Patient primigravida 6 weeks presents with severe lower abdominal pain
radiating to the shoulder.
vitals 90/50
HR 118
US: No intrauterine pregnancy
What is the most appropriate next step
A-Laparoscopy
B-Methotrexate
C-Prostaglandin
215
196. Patient eclamptic at 34 weeks have been resuscitated and stabilized what to do
next?
A-Deliver
B-Wait till 37 weeks then deliver
Answer: A
Speciality: OBGYN
Explanation:
216
197. Patient delivered a baby boy 4.5 KG, with successful delivery of the placenta,
after that she had perfuse bleeding which of the fowling is the most appropriate step?
A-Assess uterine contraction
B-Blood for coagulation profile
C-Blood for complete blood count
Answer: A
Speciality: OBGYN
Explanation:
217
198. A 30 Y.O male presented with hematemesis
See report:
Upper endoscopy: Fundal varices controlled by injection sclerotherapy
Imaging: Splenomegaly, normal portal vein, thrombosed splenic vein
What is the most appropriate management ?
A-Portocaval shunt
B-Splenorenal shunt
C- Sengstaken-blakemore tube
D- splenectomy
Answer: D
Speciality: Surgery
Explanation:
218
199. Female pregnant with SOB and dyspnea exam reveals lower leg swelling what
to do?
A. V/Q scan
B. CXR
C. Lower leg compressive US
D. CTPA
Answer: C
Speciality: Medicine
Explanation:
219
200. Streptokinase antidote:
A. Factor VIII concentrate
B. Vitamin K
C. Protamine
D. Aminocarpoic acid
Answer: D
Speciality: Medicine
Explanation:
220
201. 34 yo female medically free presented with painless non-tender mobile firm
mass in the left lobe of thyroid, right is completely normal
She have no symptoms and no history of radiation
What to do next:
A. Thyroid Tc99 scan
B. FNA
C. T3 level
D. TSH
Answer: D
Speciality: Medicine
Explanation:
221
202. Female diagnosed with bilateral DVTs and she has history of 2 still births
Labs: Prolonged APTT (no other labs provided)
What is the diagnosis:
A. Antiphospholipid syndrome
B. Protein S deficiency
C. Protein C deficiency
Answer: A
Speciality: Medicine
Explanation:
222
203. An 8 year old girl with persistent and continuous diarrhea, she drinks 3 pints of
goat milk per day and is a fussy eater. Which of the following conditions explain her
presentation?
Labs:
Hgb Low
MCV high
MCHC high
A- *psycological* Deprivation
B- Giardiasis Parasitic Infection
C- Aplastic Anaemia
D- Hypothyroidism
Answer: A
Speciality: Pediatric
Explanation:
223
204. 46 yo female known to have varicose veins for 6 years presented with bilateral
leg swelling and pain after prolonged standing
No hx of claudication
After examination there’s various veins in the great sephonus vein territory
What is the investigation of choice:
A. CT venography
B. Venous duplex
C. venous plethysmography
D. No investigation required
Answer: B
Speciality: surgery
Explanation:
224
205. Eldry k/c HTN DM smoker since 40 years Underwent thromplysis for stenosis in
hospital , HA1c = 7 What the risk condition could be happened during his admission!?
A-stroke
B-bleeding
C-PE
D -MI
Answer: A
Speciality: Medicine
Explanation:
Another recall
225
206. A case of suspected MER-COV, the patient asked you not to report it as he is
afraid he may lose his job. An infection control senior is waiting for results so that he
can send it to the ministry of health. What is the most appropriate action?
A- Confirm the case then report
B- Report as suspected case within 24 hours
C- Treat the patient then report
D- Respect the paitent’s wishes and do not report
Answer: B
Speciality: ethics
Explanation: ministry of health
226
207. Neurogenic shock case Best initial treatment:
- iv fluids
- O- blood
- steroid
Answer: A
Speciality: medicine
Explanation: by exclusion:)
227
208. 50 y/o female present with uterine spotting, no history of wt loss, most probable
cause:
Endometrial Hyperplasia
Endometrial hypoplesia
Uterine cancer
Answer: A
Speciality: OBGYN
Explanation:
There is No Hx of weight loss.
228
209. 45 y\o female medically free present with flank pain, fever what is the
treatment:
A-Ciprofloxacin
B-Ceftriaxone
Answer: A
Speciality: medicine
Explanation:
Case of UTI
229
210. adulescent not received varicella vaccine. when to give
A. give the 1set dose and the second after 6 month
B. give on dose only
C. no Need for vaccine
Answer: A
Speciality: miscellaneous
Explanation:
230
211. highly suspected cancer in colon adenoma
A. tubular
B. tubulovillous
C. villous
Answer: C
Speciality: medicine
Explanation:
231
212. preterm baby present with hydronephrosis and distended bladder
A. prune belly syndrome
B. obstructed urethrovasical junction
Answer: B
Speciality: pedia
Explanation:
232
233
213. scenario of typical symptoms of MI with st elevation in AVL I v2 and V5
A. anterior
B. lateral
C. inferior
D. posterior
Answer: B
Speciality: medicine
Explanation:
234
214. scenario of anal fistula on 7 o'clock planning for fistulotomy asking about
common are of internal able fistula
A. medial posterior
B. medial anterior
C. lateral
D. supination
Answer: A
Speciality: surgery
Explanation: schwartz
235
215. pediatric pt with respirator infection treated 6week ago withe antibiotics .and
now antibiotics given developed abdominal pain and watary diarrhea which type of
organism
A. colstridiodes difficele
B. colstridiodes perfringens
C. sterptococcus pneumonia
D. staphlycoccau auras
Answer: A
Speciality: pedia
Explanation:
236
237
216. pediatric with thyroglossal cyst management
A. observation
B. aspiration
C. surgical excision
Answer: C
Speciality: pedia
Explanation:
238
217. asymptomatic elderly presented for abdominal pain evaluation no fever no
wight loss no night sweating chest x-ray showed 1cm lesion biopcy showed
noncaseaiting granuloma
what is that appropriate management?
A. observation
B. follow up after or every 6 month by CT chest
C. start anti TB
D. surgical excision
Answer: A
Speciality: medicine
Explanation: Sarcoidosis.
239
218. Pregnant lady,41 GA in labor on epidural analgesia, mg sulfate for pre-
eclampsia and oxytocin, CTG showed prolonged deceleration and the mother was
hypotensive, most likely cause of the CTG finding:
A. Mg sulfate
B. Oxytocin
C. Epidural analgesia
Answer: C
Speciality: OBGYN
Explanation: Dr. Wafa
📝📌Note to remember
Medications that can affect fetal heart rate: (ACOG)
- Mg Sulfate: causes minimal or reduced variability
- Epidural analgesia: causes maternal hypotension -> uteroplacental insufficiency->
late or prolonged decelerations
- Oxytocin: late or prolonged decelerations + uterine hyperstimulation
240
219. What of the following decreases the risk of preeclampsia?
A. Antibiotic
B. Aspirin
C. MgS04
Answer: B
Speciality: ob/gyn
Explanation:
241
220. What of the following decreases the risk of eclampsia?
A- Antibiotic
B- Aspirin
C- MgS04
Answer: C
Speciality: ob/gyn
Explanation:
242
221. Patient coming from an endemic area for malaria. Later on, he was admitted to
the hospital with Heart failure. Doctor did not ask the patient about travel or
medication history. Thus, he ordered the nurse to give digoxin. Who is at fault?
A) Computerized drug prescription system
B) Nurse failed to write drug formularies
C) Pharmacist who failed to check prescription
D) Patient who did not bring his medicine pack/ report his medicines
Answer: A
Speciality: ethics
Explanation: SMLE killers
243
222. A doctor used the Foceps, place it in wrong position and injured the
stylomastoid bone, what is the result?
A.Baby cannot close his left eyes
B.Loss of taste in anterior ⅔
Answer: A
Speciality: pedia
Explanation:
244
223. Female with vaginal discharge Grey, fishy odor , smear : show clue cells,
Treatment?
A. Metronidazole
B.Topical clotrimazole
Answer: A
Speciality: ob/
Explanation: Bacterial Vaginosis
245
224. Pt old his regular doctor transfured to another hospital he’s upset and wants
medical record and referral to his regular doctor ?
A- Convince him that there’s equally compitant doctors
B- Give report and refer him
C- Refuse
D- Calm him down and tell him to come tomorrow to decide
Answer: B
Speciality: Ethics
Explanation:
246
225. Male dr wants to examine a female patient and he asked the nurse to come with
him what did the doctor practice ?
A- privacy
B- justice
C- Nonmaleficence
Answer: A
Speciality: ethics
Explanation:
247
226. Patient came with vaginal discharge she have done CS with episiotomy 10 days
ago , the obstetrician diagnosed her with UTI and described Abx , but she did not
improve then she went to another obstetrician and he found infected vaginal swab ,
What is the medical error done by the first obstetrician?
A. let the midwife assist him and depend on her
B. doctor failed to follow the surgical safety protocol in the OR
C. No communication between the 1st and 2nd obstetrician
D. Genuine different assessment or diagnosis of patient case
Answer: B
Speciality: Ethics
Explanation:
248
227. 2.5 y/o boy keeps yelling NO (and some other like this) what will you advise the
parents?
A) Ignorance
B) Counselling
C) Positive reinforcement
D) Punish the child
Answer: C
Speciality: ethics
Explanation:
249
228. PRESCHOOL CHID WILL WENT TO SPLENOECTOMY, WICH VACCINE
SHOULD HE TAKE ?
A) RUBELLA
B) VARICELLA
C) H. INFLUNZA
D) MEALESES
Answer: C
Speciality: Pedia
Explanation:
Patients with splenectomy should be vaccinated against Encapsulated bacteria, which
are S. pneumoniae, N. meningitidis, and H. influenzae
250
229. 43 YO male known case of hepatitis B infection presented with fever and
confusion, examination reveled disorientation, asteraxis and positive shifting dulness
Ascetic fluid analysis:
SAAG= 1.2
Neutrophils= 350 or 320
Culture bending
What is the most appropriate treatment?
A- IV diuretics and metronidazole
B- Lactulose enema and IV Ceftriaxone
C- High ascetic fluid aspiration and propranolol
- ???
Answer: B
Speciality: Medicine
Explanation:
Spontaneous Bacterial Peritonitis.
251
230. After ERCP for management of obstructive jaundice patient develop abdominal
pain and examination reveals distended abdomen with nick, chest, and abdominal
emphysema, which one most likely injured during this procedure:
A- esophagus
B- trachea
C- duodenum
D- stomach
Answer: C
Speciality: Surgery
Explanation: (UpToDate)
252
231. Elderly hx of DM and HTN , c/o calf pain when he walk 200 meter , which is
relieved by rest. What to advice him?
A. aerobic exercise
B. resistance exercise
C. supervised exercise program
D. non supervised exercise program
Answer: C
Speciality: Medicine
Explanation:
253
232. What is the most common site of aspiration of foreign body:
A-Trachea
B- left main bronchus
C- right main bronchus
Answer: C
Speciality: Medicine
Explanation: (Review team | Internal Medicine)
As the right bronchus is more straight than the left, so any foreign body will go there
easily.
254
233. A male patient with Ulcerative colitis had several episodes of diarrhea for 7 days.
Not associated with vomiting.
Which of the following is the best fluid to give him ?
A- Half Normal saline
B- Ringers lactate
C- Colloid
Answer: B
Speciality: Medicine
Explanation:
Diarrhea: Ringer lactate
Vomiting: Normal saline
255
234. Full term delivered healthy baby
Which vaccine should be given before discharge :
A- HBV & BCG
B- HBV& DTAP
C- MMR & BCG
D-DTAP & BCG
Answer: A
Speciality: Pediatris
Explanation:
256
235. - The most common risk factor in prostate cancer:
A- Age
Answer: A
Speciality: Surgery
Explanation: (UpToDate)
257
236. - Female 27 Years old with abdominal pain, constipation and vomiting fir 4 days,
the doctor decided to do Abdominal CT, which of the following test is indicated?
A) CBC
B) Pregnancy test
C) RFT
D) LFT
Answer: B
Speciality: ObGyn/Surgery
Explanation:
Abdominal pain and vomiting + Female in reproductive age = Pregnancy should be
excluded first.
258
237. Patient was planned to do appendectomy and in the OR the appendix was not
inflamed but the surgeon remove it as a common procedure and he didn’t recall if he
told the patient or not:
A- ask another surgon to tell the patient
B- tell the patient it as a protocol
C- consult health committee
D- don’t tell the patient
Answer: B (corrected)
Speciality: Surgery/Ethics
Explanation:
259
238. Patient presented with Greenish -brown colored ring in the Corneoscleral
junction. Complain of dysarthria, dyskinesia what is your diagnosis?
A- Hemochromatosis
B- Wilson’s disease
C- hamartoma
D- Autoimmune hepatitis
Answer: B
Speciality: Medicine
Explanation:
Kayser Fleischer ring → Wilson
260
239. Elderly patient with pelvic fracture and bleeding per rectum.
Urethrogram - retroperitoneal urethral injury.Your management
A. Foleys catheter
B. Suprapubic cystostomy
C. Laproscopic repair
Answer: B
Speciality: Surgery
Explanation:
Urethral injury = Suprapuboc catheter.
(Amboss):
261
240. Pregnant 34 weeks, she have a history of fibroids, presented with abdominal
pain and fever, no contraction felt and not in labor, whats the managment?
A- Observe
B- Myomyctomy
C- CS
D- IOL
Answer: A
Specialty: ObGyn
Explanation: UpToDate
Note: Fever can be a presentation of fibroid.
262
241. patient with a history of radiofrequency ablation is at risk for hypothyroidism. He
comes for a checkup and his labs show a TSH of 5 (Normal is 0.5 - 5.0). What will you
do to establish a diagnosis
A. Repeat TSH in 4 weeks
B. Measure T4 levels
C. Thyroid scan
D. Thyroid US
Answer: B
Speciality: Medicine
Explanation: (UpToDate)
263
264
242. Patient on TPN, what’s a common metablic abnormality?
A- Hypokalemia
B- Hypocalcemia
C- Hypophosphatemia
Answer: C
Speciality: Medicine
Explanation: (UpToDate)
265
243. Polyhidraminos, what a possible association?
A- trisomy 21
B- Renal agenisis
C- IUGR
Answer: A
Speciality: Pediatrics/ObGyn
Explanation: (UpToDate)
266
244. what’s a common cause for polyhidraminos?
A- Uncontrolled diabetes
Answer: A
Speciality: ObGyn
Explanation: (Dr. Wafa’a)
267
245. 55 years old female came to the clinic for regular follow up. 6 months age she
was diagnosed with atrial fibrillation. Now she is asymptotic and has no palpitations,
her medical history is significant for transit ischemic attack and hypertension. She is on
Warfarin, Beta blockers, and statins. What is the most appropriate management for
her?
A- Discontinue warfarin
B- Continue warfarin
C- Continue warfarin and start aspirin
D- Discontinue warfarin and start clopidogrel
Answer: B
Speciality: Medicine
Explanation:
268
246. Patient with 2 perianal fistulas, GI symptoms (but not diagnesd with crohn), what
a high diagnostic tool?
A- Colonscopy
Answer: A
Speciality: Surgery
Explanation:
This patient presented with GI symptoms, and crohn's diseases should be
invastigated.
269
247. inhalation burn injury, whats the most important to protect?
A- Airway
B- Breathing
C- Circulation
Answer: A
Speciality: Emergency
Explanation: ABCD protocol.
(BMJ):
270
248. 35 yrs old pregnant patient, she is currently a smoker, has history of subfertility
and family history of DM, what is the most risk for this current pregnancy?
A- Age
B- Smoking
C- history of subfertility
D- FH DM
Answer: A
Speciality: ObGyn
Explanation:
- 35 and more is the risk factor.
- Less than 35, smoking will be the risk factor.
271
249. trichomoniasis scenario how to treat the husband?
A- Metronidazole
B- No need
Answer: A
Speciality: ObGyn
Explanation: (Amboss)
272
250. CTG of prolonged decelration, what ms the possible cause?
A- Epidural analgesia
Answer: A
Speciality: ObGyn
Explanation: (Dr. Wafa’a)
Medications that can affect fetal heart rate: (ACOG)
- Mg Sulfate: causes minimal or reduced variability
- Epidural analgesia: causes maternal hypotension -> uteroplacental insufficiency->
late or prolonged decelerations
- Oxytocin: late or prolonged decelerations + uterine hyperstimulation
273
251. HF patient already on bb, acei, furusemide now has dyspnea and basal crakles,
what to add?
A- spironolactone
B- digoxcin
C- ccd
Answer: A
Speciality: Medicine
Explanation:
HF:
(ABCD)
A= ACEI or ARB (Given)
B= BB (Given)
C= Aldosterone antagonist ( K sparing diuretics ) - spironolactone (To be given)
D= Loop Diuretics (Given)
274
252. Asthma and allergic rhinitis came to clinic after she was discharged 2 weeks ago
from admissions with asthma exacerbation. She has uncontrolled symptoms of asthma
and night symptoms she is on SABA, ICS, LABA,what to give medications?
A. leukotrines inhibitors
B. oral cortisone
C. oral salbutamol
Answer: A
Speciality: Medicine
Explanation:
Asthma management stepwise:
275
253. 41 year old lady with symptoms of allergic rhinitis (runny nose, tearing during fall
season ) and she is taking meds for that, but she is complaining of weezing and SOB
and cough during fall season affecting her daily life so much, she was given saba and
improved significantly what will you give here with SABA?
A- ICS
B- ICS and LABA
C- recommed daily oral anti histamine
D- Montelukast
Answer: A
Speciality: Medicine
Explanation:
276
254. 27 years old man came to the clinic with family history of sister with adult
polycystic kidney disease , how u will screen him?
A- US
B- polycystin-1 antibodies level
C- Biopsy
Answer: A
Speciality: Medicine
Explanation: (UpToDate)
277
255. pediatric taking chemotherapy, contacted a child with chickenpox and now he
has it, how to treat?
A- Acyclovir
B- stop chemo sessions
C- varicella immunoglovlin
Answer: A
Speciality: Medicine
Explanation: (Amboss)
278
256. Pediatric patient came with fever and sore throat, the tonsils were congested
and he had papular lesion with erythematous base on his mouth and gingivitis:
A. HSV
B. EBV
C. Coxsackie
Answer: A
Speciality: Pediatrics
Explanation: (Amboss)
279
257. Rashes after hot shower
A. measles
B. rheumatic fever
Answer: B
Speciality: Medicine
Explanation: (Dr.Safdar)
Better recall:
9 yo child has knee pain that resolved than develped ankle pain. He
Has skin rash after warm showers:
A- measls
B- chickenpox
C- juvenile rhemotoid arthritis
D- rheumatic fever
Answer: D
280
258. Unvacinated child came with lymph node swelling and fever, when they tried to
take culture there was bleeding from the throat, what is the most likely diagnosis ?
A- Diphtheria
B- H.Influenza
C- Streptococcus pneumo
Answer: A
Speciality: Pediatrics.
Explanation: (Amboss)
281
259. patient with chronic constipation, she mentioned that she needs to push the
vagina backwards to relieve the constipation, what is the best management ?
A- Removal of the enterocele
B- Anterior Colporrhaphy
C- posterior Colporrhaphy
D- I don't remember the 4th option
Answer: C
Speciality: ObGyn
Explanation:
Constipation = Rectocele = Posterior Colporrhaphy
282
260. Patient has grandiose delusions and hallucinations, what does he have ?
A- Neurosis
B- Psychosis
Answer: B
Speciality: Psychiatry
Explanation:
283
261. patient diagnosed with schizophrenia and started treatment , what is true about
his prognosis ?
A- 5% have remission
B- Most will have remission?
C- 70% will have good quality of life
D- 33% will reduction of symptoms
Answer: D
Speciality: Psychiatry
Explanation:
This is the closest answer of what I found :)
284
262. 3 month Preterm baby has a chronic lung disease, both parents are smokers, he
was sleeping in with his parents in there bed, parents brought him dead to the ER,
what is the most likely cause?
A- Sudden lung distress syndrome
B- Sudden infant death syndrome
Answer: A
Speciality: Pediatrics
Explanation:
By exclusion, and as SIDS defined as an unexplained death of an apparently healthy
infant.
And in our case, this baby is not a healthy baby unfortunately.
285
263. pregnant patient at 37 weeks gestation, known case of RH immunization, fetus
found to have anemia, what is the management?
A- Delivery of the baby
B- Give blood to the fetus ( not written like this)
Answer: A
Speciality: ObGyn/Pediatrics
Explanation:
This baby can be delivered safely Inshaallah.
286
264. Patient had a fracture and is being treated, he developed severe pain and
swelling, what which of the following signs, indicates the earliest damage?
A- Decrease pulse
B- Parasthesia
C- Discoloration
C- Foot drop
Answer: B
Speciality: Surgery
Explanation:
287
265. Dm + htn + ckd came with hight bp low k and high crea what the immediate
step in management
A- Diyalisis
B- NAHcO3
C- Calcium gluconate
D- Insuline
Answer: C
Speciality:
Explanation:
I think there is some wrong information in this recall, as K level is high in CKD which
means we need to protect the heart as a 1st step in the management by giving
Calcium Gluconate.
288
266. 22 y female admitted with pulmonary hemorrhage and golomerunephratis hx of
sinisitis and numbness in right upper limb and left lower limb
Dx :
A- Giant cell arthritis
B- Hsp
C- Granulomatosis with polyangitis
Answer: C
Speciality: Medicine
Explanation: Granulomatosis with polyangiitis - AMBOSS
289
267. 60 y post tb bronchiectasis with massive hemopttysis and crepitation in right
scapular area wof initial management
A/sedation
B/positional right lateral decubitus
Answer: B
Speciality: medicine
Explanation: Evaluation and management of life-threatening hemoptysis - UpToDate
290
268. After circumcision had profuse bleeding,
Labs🧪
:arrow_up: PTT
INR: i think 1. Something but I don’t remember
what to do ?
A- tell parents that’s fine
B- blood transfusion
C- packing
Answer: C
Speciality: Pedia
Explanation: Complications of circumcision - UpToDate
291
269. A 58 years old patient known case of CKD, labs showed iron deficiency anaemia,
What is the best way to manage ?
A-start iron replacement
B-start erythropoietin
C-start him folic acid
D-start iron then erythropoietin after correction of iron level
Answer: D
Speciality: Medicine
Explanation: Treatment of anemia in nondialysis chronic kidney disease - UpToDate
292
270. 61 years old male patient came for routine check up, his Echo showed
constrictive LV hypertrophy and severely stenotic aortic valve.
His ejection fraction is Normal.
ECG is unremarkable.
He is symptomatic.
What is the most appropriate next step ?
A-ballon valvuplasty
B-ace inhibitor
C-AV replacement
D-follow up in 6 months
Answer:
Speciality: Medicine
Explanation: Aortic stenosis - Treatment algorithm | BMJ Best Practice
293
271. Patient on renal disease and decreased GFR and need to dialysis in next year,
What is the risk of death for this patient ?
A-renal failure
B-coagulpathy
C-cardiovascular disease
Answer: C
Speciality: Medicine
Explanation:Overview of the management of chronic kidney disease in adults -
UpToDate
294
272. patient comes with cough, hemoptysis What is the initial action?
a. Acid fast bacilli
b. Chest X-Ray
c. Isolation in a negative pressure room
Answer: C
Speciality: Medicine
Explanation:Tuberculosis transmission and control in health care settings - UpToDate
295
273. Patient with ascites, paracentesis showed high saag >1.1
What is the diagnosis?
A-TB
B-nephrotic syndrome
C-liver cirrhosis
Answer: C
Speciality: Medicine
Explanation: Ascites - AMBOSS
296
297
274. Pregnant 🤰 in labour was induced by oxytocin, CTG showing late deceleration
with picture,
What to do to reverse condition ?
A- change mother position to sleep supine
B- give epidural anaesthesia
C- weird word CEASING oxytocin but i chose it
Answer: C
Speciality: OGGYNE
Explanation: Dr Wafa (page 110)
298
275. 39years old patient with symptoms of SLE, arthritis and malar rash,
what medication should be started ?
A-Hydroxychloroquine and Mycophenolate mofetil
B-Hydroxychloroquine and Methotrexate
C-Hydroxychloroquine and azathioprine
D-Hydroxychloroquine and steroid
Answer: D
Speciality: Medicine
Explanation:Systemic lupus erythematosus - Treatment algorithm | BMJ Best Practice
299
276. A 66 years old patient did CT angio 4 days ago, he cMe today complaining of
oliguria and abnormal renal function,
What is he having ?
A-UTI
B-Renal cortical necrosis
C-Acute tubular necrosis
D-Renal stone
Answer: C
Speciality: Medicine
Explanation: Acute kidney injury - AMBOSS
300
277. A 50 years old female no past surgical and medical history, presented with
complaints of urine incontinence with coughing and sneezing,
Most appropriate management ?
A-urethral ring
B-coloporrhaphy
C-pelvic floor exercise
D-burch procedure
Answer: D (corrected)
Speciality: OBGYNE
Explanation:Most appropriate means best
The best in stress incontinence is midurethral sling, then burch procedure
301
278. Trauma Pt with extra peritoneal bladder injury?
A-Suprapubic catheter
B-Urgent exploration and repair
C-Catheter repair and assess after 2 weeks
D-Catheter us after 2 weeks
Answer: C
Speciality: Surgery
Explanation: Dr Hamzah
302
279. PROM ,, 34 weeks, What to give ?
A- Tocolytics
B- steroid
C- antibiotic
D- irrelevant
303
280. 45 years old female with retrosternal chest pain, dysphasia to liquids more the
solids.
What is best diagnostic method ?
A-barium swallow
B-oesophageal sphincter manometry
C-endoscopy
Answer: B
Speciality: medicine
Explanation:Achalasia: Pathogenesis, clinical manifestations, and diagnosis -
UpToDate
304
281. 56 year old male with long standing heartburn, did endoscopy and showed
barrette esophagus with low dysplasia, what is the initial ?
A Esophageal resection
B Pantoprazole
C Renitadine
D Sucralfate
Answer: B
Speciality: Medicine
Explanation:Barrett's esophagus: Surveillance and management - UpToDate
305
282. Child presented with sandpaper like rash , he had URTI a week ago.
What's the causative organism?
A. Parvovirus B19
B. Streptococcus pyogenes
C. Streptococcus pneumonia
Answer: B
Speciality: Pedia
Explanation: Scarlet fever - AMBOSS
306
283. Urine incontinence while running, what is the test to confirm diagnosis ?
A- Uroflowmetry
B- Urodynamic test
C- cough test
D- tip test
Answer: C
Speciality: OBGYNE
Explanation: Stress incontinence - AMBOSS
307
284. Male patient came with scalp open wound, after 6h assault, what wound
management?
A. Secondary closure
B. Debridement with Primary closure
C. Debridement with granulation
D. Leave it for granulation
Answer: B
Speciality: Surgery
Explanation: Dr Hamzah
308
285. 4 years old with mid-shaft femur fracture, angulation 30 degree. What is the Best
management?
A. Closed reduction & hip spica
B. Open reduction & plate
C. Open reduction & IMN
Answer: A
Speciality: Pedia
Explanation: Femoral shaft fractures in children - UpToDate
309
286. Symptomatic (menorrhagia)What is the common fibroid type ?
A- Serosal
B- Submucosal
C- intramural
Answer: if the question mean most common type cause bleeding the answer will be B
, but if the question mean most common type the answer will be C
Speciality: OBGYNE
Explanation:
310
287. 39 years old female who has three children and completed her family diagnosed
as endometrioma which was removed 2 years ago, right ovary cyst she presented to
the clinic with mild to moderate dysmenorrhea and dyspareunia during intercourse
and chronic lower abdominal pain. Pelvic ultrasound shows: Left ovary endometrioma
cyst 6x7 in size.
A. Removal of cyst more than 10 in size
B. Aspiration of cyst content under ultrasound guidance
C. Immediate hysterectomy and salpingectomy oophorectomy
D. Removal of cyst by laparoscopic ablation of endometrioma spots.
Answer: C
Speciality: ObGyn
Explanation: (Dr. Wafa’a)
311
288. The transmission of maternal antibodies to the foetus in pregnancy 🤰 is a way of
?
A- Passive nature immunity
Answer: A
Speciality: ObGyn/Pediatrics
Explanation:
312
289. Ride tricycle ,dress him self with help , point to the colour when he asked this is
which colour etc ,, know his age, Difficulty in drawing ✍ square ?
A- 3
B- 4
C- 5
D- 6
Answer: A
Speciality: Pediatrics
Explanation:
- The child rides tricycle at 3 (tri = 3) > Support the answer A
- The child knows own gender and sull name at 3 > Support the answer A
- The child can copy squares at 5 > So, exclude C and D
- The child can dress without help at 4 > So, exclude B
313
290. Pregnant without specifying in which gestational age ,, which vaccination she
can take ?
A- Influenza
B- Rubella
C- Dtap
Answer: A
Speciality: ObGyn
Explanation:
314
291. Patient’s family they don’t want from the doctor to share their patient’s disease
to others, you have the right to share his disease if it is?
A- reportable disease
B- Other mcqs irrelevant
Answer: A
Speciality: Misc
Explanation:
It is the interest of public safety to report certain infectious diseases to the health
authorities even without consent
(SMLE Brain | Ethics):
315
292. Case with long scenario, findings :
Decrease Serum Na+
Decrease plasma osmolality
Decrease urine osmolality
normal K
what is the cause:
A-Cushing syndrome
B-Conn’s syndrome
C-syndrome of inappropriate antidiuretic hormone
D- DI
316
293. Case of cough - conjunctivitis developed fever and Maculupapular rash all over
the body
A- Rubella
B- Paravirus
317
318
294. 5cm breast mass , thin breast :
A- I&D
B- Aspiration
Answer: A
Speciality: Surgery
Explanation:
- Normal skin? Aspiration
- Necrosis or thinning of the skin? I&D
(Amboss):
319
295. Pt have melena fatigue in history he have dental tooth extraction for which he
using iboprofen on ex pale
Labs🧪:
hb 8
maroon stool colour, management ?
A-Normal saline
B-Somthing with crytalloid flood
C-Blood transfusion
Answer: A
Speciality: Surgery
Explanation: (Dr. Abeidi)
320
296. diabetic patient with gout. Which of the following drugs aggravates his arthritis?
A: Lasix
B: Plavix
C: Aspirin
D: Metformin
321
297. treatment of brucellosis in adult?
A-Doxycycline and streptomycin
B-Doxycycline and cefriaxone
C-Rifampin and trimethprim-sulfamethaxazone
D-Rifampin and ciprofloxacin
Answer: A
Speciality: Medicine
Explanation: (Amboss)
322
298. physician was working on a research paper. Before publishing the paper,he
made some changes to some of the data and omitted other parts of his results. What
is this action referred to?
A-Falsification
B-Fabrication
C-Plagiarism
D-Near miss
Answer: A
Speciality: Misc
Explanation:
323
299. 51 years old male has 3rd degree haemorrhoids, Which of the following is the
most appropriate thing to do?
A-Colonoscopy
B-Hemorrhoidectomy
C-CT scan
Answer: A
Speciality: Surgery
Explanation:
Old age > Do Colonoscopy
324
300. baby with noisy breathing and wheezing that improves when prone and
increases when
supine. What is the diagnosis?
A. Laryngomalacia
B. Tracheomalacia
Answer: A
Speciality: Pediatrics
Explanation: (Amboss)
325
301. 16 year old , amenorrhea for 2 months and galactorrhea for 3 months , what is
the most important investigation ?
A-Prolactin
B-Progesterone
C-Oestrogen
D-LH
Answer: A
Speciality: pedia
Explanation:
326
327
302. 24 years old with history of appendectomy 5 years ago presented with
abdominal pain, distention and vomiting for 3 days, CT scan revealed signs of
intestinal obstruction & peritonitis.
What agent is contraindicated in this patient ?
A-propafol
B-nitrous oxide
C-sevoflurine
D-ketamine
Answer: B
Speciality: Surgery
Explanation:
328
303. Hyperkalemia 6.5 in CKD patient, next step ?
A. Calcium gluconate
B.insulin and salbutamol
C. Dialysis
Answer: A
Speciality: Medicine
Explanation:
329
330
304. Male presented with lateral side of right leg redness and streaks of blood
(scenario of lymphangitis) Which of the following is the antibiotic of choice ?
A-Dicloxacillin
B-Tetracycline
C-Erythromycin
D-Penicillin g
Answer: A
Speciality: Medicine
Explanation:
Option D is penicillin G, not Penicillin V
331
305. What anti diabetic medication can reduce mortality in DM patients
A. Metformin
B. Gliflozin
C. Glipizide
D. Acarbose
Answer: B
Speciality: Medicine
Explanation:
SGLT-2
332
306. Patient underwent left lower parathyroidectomy for primary hyperparathyroidism
(adenoma). He presented 4 months later with depressed mood and fatigability. Both
parathyroid hormone and calcium
were high, what is the most common cause?
A. Parathyroid hyperplasia
B. Missed adenoma
C. New adenoma
D. Parathyroid cancer
Answer: B
Speciality: Surgery
Explanation:
333
307. Wof is the most important before reduction?
A- Analgesia
B- Eaxamine pulse
Answer: B
Speciality: Surgery
Explanation: ABCD approach
334
308. 4 m dx with Down syndrome for screening wich chart :
A-Down syndrome chart
Answer: A
Speciality: Pedia
Explanation:
Similar recall:
how to assess Down syndrome growth ?
A. Stander growth chart
B. Down syndrome growth chart
335
309. Antispycotic drug ( didn't mention name of drug )was give for schizophrenia
patient what is complications your are suspected to seen :
A-MI
B-hypothermia
C-? I think seizure
Another recall :
2-Schizophrenia patient who took neuroleptic medication for her psychosis.
Which of the following is side effect of her medication?
A. Seizure
B. Akathisia
C. Hyperthermia
D. Myocardial Infraction
Answer: B
Speciality: Medicine
Explanation:
336
310. 32 YO G3P0020 admitted as a case of incomplete abortion for D&C, during
D&C, there was profuse bleeding which required vigorous curettage. After D&C she
cam to the clinic after 6 months complaining of no menstruation despite positive pre-
menstrual symptoms
Answer: C
Speciality: Ob/gyn
Explanation:
337
311. 3 YO female presented with vesicular rash in the gums, tongue, buccal mucosa
and hard palate. Upon examination the patient looked dehydrated with dry mucosal
surfaces. The lesions were painful and some of them bleeds. The mother stated her
daughter cannot eat because of pain
Answer: A
Speciality: Pedia
Explanation:
B will be unilateral vesicle on a dermatomal site (corresponding to a region with a
specific nerve supply), and usually occurs in adults as a result of viral reactivation due
to immunosuppression
338
312. Which medication for hypertension is contraindicated in asthma ?
A. Bb
B. Acei
C. Amlodipine
Answer: A
Speciality: Medicine
Explanation:
339
313. Pregnant kc of idiopathic thrombocytopenic purpura developed PPH ?
A. FFP
B. RBC
C. Platelets transfusion
D. cryoprecipitate
Answer: C
Speciality: Ob/Gyn
Explanation:
Better recall:
20 years old pregnant known case of Idiopathic Thrombocytopenic purpura. Her
delivery was complicated by post partum hemorrhage. What will you give her next?
A. Packed RBCs
B. Fresh frozen plasma
C. Cryoprecipitate
D. Platelets transfusion
340
Answer: D
341
314. 69 yo recommended vaccine
A. Pneumococcal
B. meningococcal
Answer: A
Speciality: Medicine
Explanation:
342
315. recurrent pregnancy loss in 1st. Trimester with Hx of theomebolism
A- warfarin life long
B- enoxparin life long
C- aspirin life long
D- fandiprix life long
Answer: A
Speciality: Ob/Gyn - Meidcine
Explanation:
MED talk:
APL syndrome → warfarin is anticoagulant of choice for life (changing to heparins +-
aspirin when pregnancy is planned)
343
316. 6 m/o infant present to ER with sudden apnea & color change. Mother gave
history of recurrent vomiting and multiple chest infections. On exam he looks failing
to thrive with dystonic neck posturing while crying. Which of the following considers
as high risk case for the above disease?
A- Obese baby
B- Preterm infant
C- Post pertussis infection
D- Following inguinal hernia repair
Answer: A
Speciality: Pedia
Explanation:
We are not sure that this is the right answer, some say that there was GERD in the
options, so it would be the best answer; otherwise we think preterm is the closest
answer as it is associated with more fetal complications
344
317. 10 YO female patient complaining of chronic rinorrhea , cough , conjunctivitis
by PE there was red discoloration around eyes and there puggy pale nasal mucosa
what's Ddx?
1.allergic rhinitis
2.ciliary dyskanesia
Answer: A
Speciality: Pedia
Explanation: Not competed the triad of dyskinesia
Similar recall:
child came with nasal congestion on examination and there is pale nasal polyp and
dark periorbital swelling
A- allergic rhinitis
B- rhinitis medicamentosa
C- Acute bronchitis
D- foreign body
345
318. Patient K/C of ulcer on panoprazole complaining of black tarry stool which drug
cause this side affect ?
A. pantoprazole
B. almenium hydroxylase
C. Bismuth subsalicylate
D. other medication
Answer: C
Speciality: Medicine
Explanation:
346
319. Female patient hx of depression ( symptoms ) complaining of constipation
which drug cause this side affect?
A.TCA
B. MAO
C. SSRI
Answer: A
Speciality: Psycha
Explanation:
347
320. A post Partum female her blood group is O negative and her baby O positive
what should we do?
A.not give anything
B. Give mother anti D
C. give baby anti D
Answer: B
Better recall:
A 25 year old female primigravidae just delivered and her blood group is O negative
and her baby blood group is O positive. What is the next appropriate management?
A) No Need for Anti-D
B) Give Anti-D for the mother
C) Inform the parent about the findings/result
D) Give Anti-D to both the mother and the baby
Answer: B
Speciality: Ob/Gyn
Explanation:
348
Women with large genital wart asking about management?
A.cryotherpy
B.laser cautry
Answer: A
Speciality: Ob/Gyn
Explanation:
349
321. Question about lung nodule size 8mm there was growing of nodule from
previous CT scan your management?
A. referal to thoracic surgery
B.neddle aspiration
C.follow up 3-6 m
D. Chest x.ray
Answer: A
Speciality: Surgery
Explanation: Since the nodule is growing, the best action is biopsy or resection; the
closest answer is referral to a thoracic surgeon
350
351
322. Female patient with multiple hemorrhoid at 7 9 o clock management?
A.hemorrhoidectomy
B.ligation
C.sphincrectomy
Another recall
352
323. Pregnant with came for antenatal visit had past medical hx of hypothyroidism
she was on 125 dose/ day lab showed normal TSH and T4 what your management ?
A.decrease dose
B.increase dose
C.samae management
D.no need
Answer: B
Speciality: Ob/gyn
Explanation:
353
324. Pregnant with past medical hx of AS ( aortic stenosis ) what increase mortality
to her fetus ?
A.increase stroke volume
B.increse plasma volume
Another recall
354
325. Male pediatric age did and mid stream urine culture showed more that 10
thosend colony of klepsiella (he did not mention if there is symptom or not )
management?
A.prophylaxtic antibiotics
B. No need
Answer: B
Speciality: Pedia
Explanation:
Another recall
355
326. painless hematuria definitive diagnostic test?
A. Cystoscope
Answer: A
Speciality: Surgery
Explanation:
356
327. 50 years male came for screening, found polyps and then has done
endoscopic polypectomy, what type of cells have the highest predator to
transform to malignancy?
A) Tubal
B) Villous
C) Tubulovillous
D) hyperplasia او ﺣﺎﺟﺔ ﻣﺜﻞ ﻛﺬا
Answer: B
Speciality: miscellaneous
Explanation:
357
328. 28 married woman complaining of Right iliac pain with drowsiness, on
examination she has tenderness on right iliac fossa, BP 90/40, Hemoglobin low,
WBCs normal (No Bhcg in scenario)
What’s your dx?
A) Terminal ileitis
B) Acute appendicitis
C) Ectopic pregnancy
D) Ruptured ovarian cyst
Answer: D (corrected)
Speciality: OBGYN
Explanation: ectopic pregnancy does not cause instability. However, in the given
options, ruptured ovarian cyst is the closest answer
358
329. 20 years female with vitiligo, including vulva.
Which of the following is most likely cause?
A) allergic reaction to drug
B) over use of irritant localy
C) autoimmune lymphocytes attacking melanocytes
Answer: C
Speciality:
Explanation:
359
330. 22 years old female married came for routine check up she’s medically
free, complaining of mild pain in her breast before period of 2 days and planing
to get pregnant after 2 years, asking which to do for her?
A) Pap smear
B) Ultrasound of breast
Answer: A
Speciality: OBGYN
Explanation: start cervical cancer screening at 21 ans she is 22.
360
331. Long scenario and ECG written as S1Q3T3 asking about the diagnosis .
A- the answer is PE
Answer:
Speciality:
Explanation:
361
332. 24y/o male presented after chemotherapy for the treatment of colorectal
cancer what electrolytes abnormality would you suspect?
A- hypokalemia
B- hypocalcemia
C- hyponatremia
D- hypomagnesemia
Better recall:
pt with colorectal cancer presents with abdominal pain and diarrhea for days now
experiencing generalized muscle weakness. ECG shows flattering of T wave. What are
possible electrolyte imbalance ?
1/ hypomaganesium
2/ hyponatremia
3/Hypokalemia
4/Hypocalcemia
Answer: hypokalemia
Speciality: medicine
Explanation:
362
363
333. Recommendation that at which age if you present A peanut and
eggs to the Child will decrease production of allergy? Age in month
A. 10
B. 14
C. 20
D. 24
Answer: A
Speciality: pedia
Explanation:
364
334. 17year old female, medically free, gymnast in her class, developed
breasts later and never menstruated, on examination she is tanner stage 5
but no menstruation, diagnosis?
A. hypothalamic hypogonadism
B. imperforate hymen
C. gonadal agenesis
D. testicular feminization
Answer: A
Speciality: OBGYN
Explanation:
365
335. 26 yr pt came to Er after a first episode of generalize tonic conic
seizure ,labs & examination are normal ,no neurological deficit ,what to do
next
A.EEG
B.ECG
C.LP
D.CT head
Answer: D
Speciality: Medicine
Explanation:
The first thing to do in cases of seizure without loss of consciousness is a head CT
without contrast.
366
336. female c/o came with missed abortion which drug you give her?
A-misoprostal
B- oxytocin
Answer: A
Speciality: ObGyn
Explanation: (Dr. Wafa’a)
367
337. pt with hepatitis B transmission rate
A- 0
B- 0.03
C- 0.3
D- 0.003
Answer: C
Speciality: Medicine
Explanation:
- Hepatitis B transmission rate 30% (0.3)
- Hepatitis C transmission rate 3% (0.03)
- HIV transmission rate 0.03% (0.003)
368
338. 25 yo medically free 38 GA , SVD = CTG: fetus heart rate 180 + early
deceleration , what may lead to CS?
A- fetal HR
B- early deceleration
C- patient age
Answer: A
Speciality:
Explanation:
Normal fetal heart rate is 110 - 160 anything beyond this scope is alarming.
369
339. Q- female patient k/c Renal failure + DM+ HTN, complaining of chest
pain and SOB
Lab: potassium 6 , what expected ECG changes ?
A- peaked T wave
B- peaked P wave
C- short PR interval
D- narrow QRS
Answer: A
Speciality: Medicine
Explanation:
(This is a case of hyperkalemia)
370
340. long scénario with ABG : PH 7.55 , HCO3 43, PaCO2 22, PaO2 92..?
A- uncompensated metabolic alkalosis
B- both metabolic and respiratory alkalosis
C- respiratory alkalosis with compensated metabolic
D- metabolic alkalosis with compensated respiratory
Answer: B
Speciality: Medicine
Explanation:
371
341. 54 YO female medically free came to the clinic for routine checkup. Her
cardiac examination revealed grade 4 pansystolic murmur heard best at the
apex and radiate to the axilla. She is symptomatic and the rest of her
examination is normal. What is the best next step to confirm the diagnosis?
A. Transthoracic Echo
B. Transesophageal Echo
Answer: A
Speciality: Medicine
Explanation:
Diagnosi: Mitral regurgitation
- Best next step > Transthoracic echocardiograms
- More sensitive > Transesophageal echocardiogram
372
342. esophageal varices to prevent future recurrence/bleeding?
A- BB
B- H2
C- PPi
D- No need
Answer: A
Speciality: Medicine
Explanation: (Amboss)
373
343. Case of child use Rifampicin ask about side effect?
A- Discoloration
Answer: A
Speciality: Pediatrics
Explanation:
374
344. Child (new born) with forceful vomiting, already not detected any anomaly
what the cause?
A- Pyloric stenosis
B- Necrotizing enterocolitis
C- Atresia
D- Something for the heart
Answer: A
Speciality: Pediatrics
Explanation: (Amboss)
Better recall:
6 week old infant presented with forceful vomiting, baby is is born full term to a 23 YO
primigravida uneventful pregnancy with normal vaginal delivery, on examination no
congenital anomalies were noted what is the most likely diagnosis
A) duodenal atresia
B) necrotising (something)
C) Hirschsprung's
D) pyloric stenosis
375
Answer: D
376
345. Case about Phyllodes.
A- Wide local excision
B- Simple mastectomy
Answer: D
This is most likely phyllodes
Treatment is wide local excision.
If too large > 10 cm = simple mastectomy
377
346. 56 Y.O male patient known to have an allergy to penicillin and sulfa
antibiotics presented with UTI. Which antibiotic is most appropriate?
A. Nitrofurantoin.
B. Trimethoprim/sulfamethoxazole.
C. Ampicillin.
D. Cephalexin.
Answer: A
Speciality: Medicine
Explanation:
378
347. 27 Y.O with history of confusion for 2 days and seizure for 5 hours. He is
not oriented to time, place and person. He has history of unprotected sex for 6
years. Serum cryptococcal antigen positive, what is the most appropriate next
step?
A. HIV serology.
B. SF Toxoplasmosis testing.
C. CSF Syphilis serology.
D. Herpes simplex testing
Answer: A
Speciality: Medicine
Explanation:
379
348. 25-year-old female known case of epilepsy came with generalized tonic
clonic seizure for 35 minutes and started on 20 mg IV lorazepam but did not
respond, what are you going to give her next?
A- IV Phenytoin
B- IV Phenobarbital
C- IV Steroid
D- IV Fentanyl
Answer: A
Speciality: Medicine
Explanation:
380
381
349. case about Impetigo but didn't mention diagnosis, (honey crusted lesion)
What is the organism:
A- S.aur
Answer: A
Speciality: Medicine
Explanation:
382
350. sjogren syndrome which type of RTA: + Labs: Shows low K Normal Na
A. RTA|
B. RTAII
C. RTA II
D. RTA IV
Answer: A
Speciality: Medicine
Explanation:
383
384
351. Which of the following is the best contaception in pt w a history of PE?
A-IUD
B- permenat steriilization
C-levonorgestrel
D- condom
Answer: A
Speciality: Ob/Gyn
Explanation:
IUD is the first line contraception, we can use copper IUD here
385
352. what is the management of women who came with moderate
cystorectocele?
A. Ant and post Colporrhaphy
B. post Colporrhaphy
C. Ant Colporrhaphy
D. Manchester procedure
Answer: A
Speciality: Surgery
Explanation: (Hunter group)
- Cystocele only = anterior colpoperineorrhaphy
- Rectocele only= posterior colpoperineorrhaphy
- Cystorectocele= Ant&post colpoperineorrhaphy
- Both of them + uterine prolapse= fothergill's operation ( manchester)
386
353. Paragravedous in labour, When Latent phase consedrid prolonged?
A) 2 H
B) 4 h
C) 8 h
D) 18 h
387
354. 47 year obese lady with HTN at your clinic what preventive measures to
do
1. Fasting glucose
2. Pap smear
3. Occult blood
Answer: B
Speciality: Medicine
Explanation:
The question asked about the preventive measure, not diagnostic measure; so, we
would go with pap smear as it is grade A in the USPTSF, while diabetes screening for
overweight or obese is grade B. However, if the question was asking about the
diagnostic measures, we would go with A for the diagnosis of metabolic syndrome.
388
389
355. 4-year-old boy came with midshaft femoral fracture. With 30 degree
angulation. Which of the following is the most appropriate management?
A- pavlik harness
B- Closed reduction with hip spica
C- Open reduction with IMN
D- Open reduction with fixed plate
Answer: B
Speciality: Surgery
Explanation:
390
356. - mom came to pediatrion with exclusef breast feeding and doctor advise
iron advise next month. age of baby now??
A.1
B.2
C.3
Answer: C
Speciality: Pedia
Explanation:
391
357. 4 months baby born preterm 33 weeks due to emergency cescrian section
regarding his vaccines what is the most appropriate protocol to follow?
A. Decrease the dose of vaccines and give it as scheduled
B. Give vaccines according to corrected age
C. Schedule vaccination according to chronological age
D. Create th schedule according to calculated weight
Answer: C
Speciality: Pedia
Explanation:
392
358. mother compaling that her baby was having non billous vomiting and you
noticed an olive shaped mass on the epigastric region , whats the best mx ?
A. Pyloromyotomy
B. Gastrojejunostomy
C. Balloon dilatation
D. Anastomosis
Answer: A
Speciality: Pedia
Explanation: Infantile hypertrophic pyloric stenosis - UpToDate
393
359. - Child with Down Syndrome and Complaining of Chronic Constipation
with decreased inactivity. What Screening Test to Send for the Patient?
A. hypothyroidism (or TFT i forgot)
B. Anti-Tissue Transglutaminase.
C. Stool analysis.
Answer: A
Speciality: Pedia
Explanation: Down syndrome: Clinical features and diagnosis - UpToDate
394
360. preterm labour 27 what is most important mx:
A- Atosiban
B- Abx
C- Steroid
D- Magnesium sulfate
Answer: C
Speciality: OBGYNE
Explanation: Dr Wafa
395
361. Child present with short stature, obese with pink streation which of the
following the next step ?
A- MRI brain
B- MRI adrenal
C- morning and night free cortisol
Answer: C
Speciality: Pedia
Explanation: Dr Safdar
396
362. 10-year-old boy is referred to a neurologist because of declining school
performance generalized seizures and visual disturbances that have worsened
during the past few months when he was 2 years old he had a flue-like illness
with a rash he has not received routine childhood vaccinations. Over the next
year, his condition worsens and he dies. autopsy shows diffuse encephalitis with
demyelination. Which of the following is the most likely etiology of this patient
condition?
A) cytomegalovirus
B) measles virus
C) mumps virus
D) rubella virus
Answer: B
Speciality: Pedia
Explanation: Measles: Clinical manifestations, diagnosis, treatment, and prevention -
UpToDate
397
363. Patient presented with progressive SOB and wt loss for 6 months (Clear
case of lung cancer)
pleural effusion was :
1500ml bloody with exudate effusion. What is the next step?
A. Empirical IV antibiotics
B. CT with IV contest
C. Blind pleural biopsy
D. Observation
Answer: B
Speciality: Medicine
Explanation: Overview of the initial evaluation, diagnosis, and staging of patients with
suspected lung cancer - UpToDate
398
364. COPD patient came to clinic with bilateral lower limb edema and
pulmonary
hypertension. O2 sat 86% ,PO2 8.6, PCO2 7.5, pH above normal range Which of
the
following the appropriate management to give for the patient now ?
A) start oral furosemide
B)Start oxygen therapy
C) Prednisolone therapy
Answer: B
Speciality: Medicine
Explanation: COPD exacerbations: Management - UpToDate
399
400
365. Case about vaginal dishrags grey and fishy oder and normal vaginal
PH(4.5)?
A. Streptococcus colonization
B. Bacterial vaginosis
Answer: B
Speciality: obgyn
Explanation:
401
366. Female with vaginal discharge Grey, fishy odor , smear : show clue cells,
ph>4.5,diagnosis?
A. Trichomoniasis
B. Bacterial vaginosis
C. Candida
Answer: B
Speciality: OBGYN
Explanation:
402
367. Pt diagnosis with STEMI, PCI not available, and BP 178/99, what
medication to give?
A. ASA, streptokinase, nitro,BB
B. ASA, heparin, streptokinase,BB
C. ASA,nitro,BB
Answer: A
Speciality: Medicine
Explanation: if patient has inferior MI do not give nitro
403
368. 40 or 50 female k/c of dm Came with retrosternal pain at rest for 3 hours
ECG done T inversion in (V2 v3 not sure about leads)
Labs :
high troponin 10
What is your diagnosis
A-Stable angina
B-Unstable angina
C-St elevation infraction
D- Non St elevation infraction
Answer: D
Speciality: Medicine
Explanation:
404
369. Patient female and young in her 30s with all meningitis s&s and CT done
with area of low attenuation what is the causative organism:
A- HSV
B- pneumococcus
C- N meningitis
Answer: A
Speciality: Medicine
Explanation:
405
370. Pt presented to ER with signs of peritonitis dx as perforated appendicitis
managed surgically, post op day 4 he developed vomiting and abdominal pain
with obstipation
xray showed multiple air fluids level in small bowl
Pt is vitally stable
Most likely dx is?
A. Paralytic ilues
Answer: A
Speciality: Surgery
Explanation:
406
371. Pt with poor apetite, fatigue, low self-esteemand feeling hopelessness.
For 2 yrs. Dx?
A. Depressive disorder
B. dysthymia
C. Mood ..
Answer: B
Speciality: PSYCHA
Explanation:
407
372. 23 Y.O. male patient K/C of UC presented with 7-8 bouts of bloody
diarrhea per day. Associated with fever and joint pain. O~E patient looks ill and
in pain. What is the best next step in management?
A. Sulfasalazine
B. IV methylprednisolone
C. Oral 5-ASA
D. Azathioprine
Answer: B
Speciality: Medicine
Explanation: ام اﻟﻘﺮى
408
409
373. Q about lesion in labia majora in post menopausal female showed
dysplasia (carcinoma in situ I think) what to do:
A. Steroid cream
B. Local excision
C. Vulvectomy
D. Repeat test after 6 months
Answer: B
Speciality: OBGYN
Explanation:
410
411
374. 16 YO came to ER after vomiting once with blood , she had recurrent N/V
before her period in the last time there was slight blood with vomiting , after 4-6
hours they mention also ,all labs and exams are normal what you should so?
A- admit her for observation
B- reassure and ask to come if it recur again
C- prepare for urgent EGD
D- Discharge
Answer: B
Speciality: Surgery
Explanation: No active bleeding + patient is stable.
412
413
375. Primary infertility with scanty irregular cycle and very high FSH, most
important work up?
A-FSH/LH in urine I think
B-CBC
C-Pelvic US
D-chromosomal analysis
Answer: D
Speciality: Ob/Gyn
Explanation: in the Q Pelvic us NOT Transvaginal US So i think Karyotype is best.
414
415
376. Most appropriate test to diagnose endometriosis
A. US
B. Diagnostic laparoscopy
416
417
377. A 42w pregnant come to antenatal care. Head well fixed in pelvis and
cervix is favorable. Most appropriate management?
A. CTG
B. IOL
C. CS
Answer: B
Speciality: Ob/Gyn
Explanation:
418
378. Pregnant complain of recurrent stillbirth and positive pregnancy test which
vaccine can give her!
A- Rubella
B- Influenza
C- Measles
D- Varicella
Answer: B
Speciality: Ob/Gyn
Explanation: Rubella vaccine is contraindicated during pregnancy
419
420
379. Advantages of mediolateral episiotomy over midline episiotomy:
A. Less rectal tear
Answer: A
Speciality: Ob/Gyn
Explanation: Beckmann and Ling’s Obstetrics book
421
380. Female patient using warfarin ). What contraception is most appropriate
for her?
a) Pop
b) Depoprovira
c) IUCD
d) OCP
Answer: C
Speciality: Ob/Gyn
Explanation:
422
381. child with pica, pale, low socioeconomic state , low haemoglobin, hand
lead was 2 , iron profile was normal, x-ray report dense metaphyseal bands ,
what is the treatment?
A. vit-K
B. D-penicillamine
C. Iron
Answer: B
Speciality: Pedia
Explanation:
423
382. Which of the following is the indication of cervical cerclage?
A. Cervical length less than 30
B. Cervical length less than 35
C. Cervical length less than 25
D. Cervical length less than 20
Answer: C
Speciality: Ob/Gyn
Explanation:
424
383. Patient with long term history of DM type 1 in 12 weeks of gestation.
HbA1C 12. Which of the following complication is most likely to happen?
A. Preeclampsia
B. Polyhydroamnios
C. Congenital malformation
D. IUGR
Answer: C
Speciality: Ob/Gyn
Explanation:
425
384. G2P0 20 weeks of gestation, with cervical incompetence (cervix length
30mm), whats the most appropriate management?
A. Cervical cerclage
B. Strict bed rest
C. Progesterone supplementation
Answer: C
Speciality: ob/gyn
Explanation:
426
385. woman with spotting scanty blood after intercourse. She had a history of
warts on vulva with cryotherapy done 2 yrs back what is the site of the bleeding?
A. vulva
B. vagina
C. Uterine cervix
D. Uterine body
Answer: C
Speciality: ob/gyn
Explanation:
427
386. 55 y.o Patient k/c smoker only. Presents with signs and symptoms of MI.
ECG shows lead II,III,AVF depression. Hist vitals are: BP: 150/92 Pulse: 99 Which
of the following is the most considerable risk factor for MI in this case?
A. HTN
B. Smoking
C. Age
Answer: B
Speciality: Medicine
Explanation: Um Alquraa
428
387. A pregnant woman GA 30 weeks with preterm labour was given a
Tocolytic. What is the rationale behind giving tocolytics ?
A- To Delay the delivery until 37 weeks
B- To maximize the effect of steroids
C. To prevent PROM
Answer: B
Speciality: ob/gyn
Explanation:
429
388. What is the medication for seizure in pregnancy?
a- Diazepine
b- Phenytoin
c- Magnesium sulfate
d- Lorazepam
Answer: C
Speciality: ob/gyn
Explanation:
430
389. Female found that she have UTI, the patient has penicillin resistance what
drug is contraindicated for this patient?
A. Pipracillin tazopactam
B. Cipro
C. Ceftriaxon
Answer: A
Speciality: Medicine
Explanation:
431
390. 45-year-old women medically free with no personal or family history of
cancer. Asking about when to to start colon cancer screening?
A- no need for screening for her case
B- Start now and every 5 years
C- Start at 50 years with annual colonoscopy
D- Strat at 50 with annual Fecal occult blood
Answer: D
Speciality: Surgery
Explanation:
432
391. Case of Depressed patient and the doctor will prescribe a drug for her.
asking which reuptake inhibitors should be prescribed?
A- DOPA
B- GABA
C- serotonin
D- Acetylcholine
Answer: C
Speciality: Psychiatry
Explanation:
433
392. Case of saddle PE Sudden SOB and the patient was unstable, BP: 90/50,
HR: 120, O2 saturation: 89%. CTA showed massive saddle pulmonary embolism.
Which of the following is the most appropriate management?
A- LMWH
B- Aspirin
C- thrombolytic
D- warfarin
Answer: C
Speciality: Medicine
Explanation:
434
393. Patent with behavioral change and become aggressive and violent?
A. low serotonin
B. high serotonin
C. low endorphin
D. high endorphin
Answer: A
Speciality: Misc
Explanation:
435
394. 12 y/o presented with chronic diarrhea for 3 months, sometimes it’s
bloody and mucus. Abdominal pain , bloating. History weight loss and joint pain
and ache. Exam : pale, abdomen: soft, lax and no tenderness
Temperature was 38.5.. CRP 1.5? lab low Hb and high urea
Diagnosis?
1- UC
2- Crohn’s disease
3- Celiac disease
4- Chronic dysentery
Answer:
Speciality:
Explanation:
436
395. Patient presented with DVT what regimen to use:
A- Aspirin 61 mg
B- Enoxaparin 40 mg SC
C- Fondaparinux 20 mg
D- Hepatin 10,000 U IV
Answer: D
Speciality: Medicine
Explanation:
437
396. Child came with painfull red swollen hemiscrotum. On examination, mass
was palpated with -ve cough impulse. The mass was tender and extended to the
inguinal area. Left testes cannot be palpable. Which of the following is the most
likely diagnosis?
A- testicular torsion
B- Epidydomorchitis
C- Incarcerated inguinal hernia
D- Testicular appendicular tortion
Answer: C
Speciality: Surgery
Explanation:
438
397. 63 years old male came with rectal bleeding with unintentional weight
loss, he was pale, on PE empty rectum and no masses were identified but
bleeding with fresh blood was noted. Which of the following is most appropriate
next step?
A- proctoscopy
B- colonoscopy
C- UGI Endoscopy
D- fecal occult blood
Answer: B
Speciality: Medicine
Explanation:
439
398. Male patient complains of episode of hematemesis. normal past medical
history Labs show mild anemia ,All over labs are normal except elevated urea
level in blood?
A-Mallory Weiss syndrome
B-erosive gastritis
C-PUD
Answer: C
Speciality: Medicine
Explanation:
440
399. Pt with renal colic what to do next :
A.MRI
B.abdominal Ct
C.Us
D.X ray KUB
Answer: B
Speciality: Medicine/Surgery
Explanation:
441
400. The baby heart auscultation on the mother umbilicus ,,which is the
presentation?
A-shoulder
B-face
C-breech
Answer: A
Speciality: ObGyn
Explanation:
I got this question in my exam, there was fetal kicks in the lower abdomen +
ballotment, in that case the answer would be breech
442
401. Elderly patient DM and chronic smoker came to be evaluated for 2 days
history of sudden left arm weakness, and there was no pain that resolved
spontaneously. On examination, there wasn’t carotid bruit. All PE was normal
apart from cardiac murmurs. Which of the following is the most appropriate
initial imaging?
A ⁃ TEE
B ⁃ carotid duplex US
C ⁃ Carotid CTA
D ⁃ Brain MRI
Answer: D
Speciality: Medicine
Explanation:
443
402. Case of A. Fib on warfarin the on follow up the INR was 6.5, she isn’t on
high risk of bleeding nor undergoing surgery. Which of the following would be
appropriate step after holding the warfarin?
A- vit K
B- FFP
C- Reassess after 2 day
D- FFP and cryoprecipitate
Answer: C
Speciality: Medicine
Explanation:
444
403. Case of patients with right unilateral lower limb weakness and loss of
sensation. Which artery is most likely affected?
A ⁃ Left MCA
B ⁃ Left ACA
C ⁃ Basilar artery
D ⁃ Vertebral artery
Answer: B
Speciality: Medicine
Explanation:
445
404. Malaria officer asked the municipality (amana) "something about
eradication of mosquitoes".
Which of the following mosquitoes should be targeted:
A. Curlex
B. Anopheles
C. Aedes aegypti
D. Aedes albopictus
Answer: B
Speciality: Misc
Explanation:
446
405. A mother of a 3 month old baby Telling the doctor she’s going crazy She
thinks that there’s a snake near her baby. She said that she didn’t see the snake
and she knows it’s not true. She check on baby crib 50 times a day What’s the
most likely diagnosis
A. Delusional
B. Hallucination
C. Postpartum psychosis
D. Obsession
Answer: D
Speciality: Psychiatry
Explanation: (Psych-Smle Channel)
447
448
406. 30 years old pregnant women by in vitro fertilization (IVF) and diagnosed
with ectopic pregnancy and scheduled of laproscopic removal of the prgnency.
On US imaging: 4 cm tubal pregnancy on the right, and hydrosalpinx on the left.
What is the management?
A. Salpingotomy on the right only
B. Salpingectomyontherightonly
C. Removal of both tubes(Bilateral tuboectomy)
D. Hysterosalpingography with contrast
Answer: C
Speciality: ObGyn
Explanation: (Dr. Wafa’a)
449
450
407. Pregnant lady 39W her routine BP throughout the pregnance was 120/80
(normal) then suddenly became 150/90 what is the diagnosis?
A. Eclampsia
B. Gestational hypertension
C. Chronic hypertension
D. Superimposed hypertension
Answer: B
Speciality: ObGyn
Explanation: (Dr. Wafa’a)
451
408. Pregnant 34 week with preeclampsia, high BP 170/100 what to give ?
A. Stabilize, MgSo4 and deliver
B. Stabilize, corticosteroids and deliver
Answer: A
Speciality: ObGyn
Explanation: (Dr. Wafa’a)
452
409. At what gestational age we start diabetes screening in a pregnant woman?
A. 4 weeks
B. 12 weeks
C. 24 weeks
D. 34 weeks
Answer: C
Speciality: ObGyn
Explanation: (Dr. Wafa’a)
453
410. Ventose delivery below periosteum?
A. Chignon
B. Cephalohematoma
C. Caput succ
Answer: B
Speciality: ObGyn/Pediatrics
Explanation:
454
411. Patient came for routine f/u and did CT scan,accidentally found 1 cm lung
nodule , he’s smoker. The mass was 1cm with sharp borders. No symptoms
A-Biopsy
B-Surgery
C-No follow up
D-Follow up ct 3 to 6 mo
Answer: D
Speciality: Surgery
Explanation: (UpToDate)
455
Note:
Our patient has > 8 mm nodule, with low malignancy risk, so we will go with ct
in 3 - 6 months.
456
412. Patient knowing for DM And HTN and history of right leg pain increased
by exertion, on examination absent popliteal pulse on right leg
Which one of the following indicates acute limb ischemia:
A-intermittent claudication
B-rest pain
C-scar for iliofemoral bypass in left leg
D-swelling
Answer: B
Speciality: Surgery
Explanation:
457
413. Pregnant 28 weeks came with bilateral breast mass it was movable and the
size was3x4 cm, most appropriate next step?
A- reassurance and follow up after delivery
B-bilateral breast ultrasound
C-Bilateral breast mammogram
D-MRI
Answer: B
Speciality: ObGyn
Explanation:
Breast lumps detected during pregnancy are generally benign and reflect
fibroadenoma, lactating adenoma, cysts, infarction of the breast or galactocele.
Although rare, the possibility of breast cancer must also be considered to avoid any
delays in diagnosis. After patient questioning and clinical examination, the first
imaging modality to use is ultrasound.
458
414. Typical case of hemolytic anemia and blood film show spherocytosis,Hb:8,
MCV MCHC normal range, WBC: 3, PLT: 500, LDH AST ALT high, low
haptoglobin most appropriate treatment?
A-blood transfusion
B-steroids
C-hydroxyurea
Answer: B
Speciality: Medicine
Explanation:
Autoimmune hemolytic anemia.
459
415. 78 YO post-op contaminated surgery. Admitted to ICU and given 6 L of
crystalloid IVF and 1 PRBC in the last 12 h. Broad spectrum antibiotics were
given. , CVP is 40”over fluid treatet = no hypoVoluimia” 8-12
Vitals:
BP 70/45
HR: 125
T: 38.6
O2: 93%
Labs: WBC 8 Most appropriate IV intervention:
A-albumin
B-2 L crystalloid fluid
C-norepinephrine
C-frusemide
Answer: C
Speciality: Medicine
Explanation:
460
416. 31w pregnant lady complains of small and slow vaginal bleeding, shes a
confirmed case of low lying placenta,
A-expectant management
B- Biophysical profile
C- CTG
D- Emergency delivery
Answer: C
Speciality: ObGyn
Explanation:
Antepartum hemorrhage:
1st, we do US to exclude placenta previa. In this case, the patient is already
diagnosed, so next is CTG.
461
417. Full term baby Born by CS developed Cyanosis and ribs retraction with
grunting, Preductal O2 was higher than Postductal O2.
What to do Next?
A-Surfactant
B-inhaled Nitrous oxide
C- forgot
D- forgot
Answer: B
Speciality: Pediatrics
Explanation: (Dr. Safdar)
462
418. 25 Y/O pt presented with RUQ pain, Night sweats, malaise, fever US
finding: Rt lobe Liver abscess, Hypoechoic CT Finding: Thick walled abscess
without septations.
What is the most appropriate Next step?
A-Ceftriaxone
B-Metronidazole
C-Surgical drainage
D-Percutaneus drainage
Answer: B
Speciality: Surgery
Explanation:
463
464
465